Article Type
Changed
Fri, 01/18/2019 - 14:43

Introduction

Biliary tract carcinoma (BTC) is the term for a heterogeneous group of rare gastrointestinal malignancies1 that includes both carcinoma arising from the gallbladder and cholangiocarcinoma, which refers to diverse aggressive epithelial cancers involving the intrahepatic, perihilar, and distal biliary tree.1–3 In this article, we review the epidemiology, clinical features, and diagnostic approach to BTC, with a focus on current evidence-based treatment strategies for localized, locally advanced, and metastatic BTC.

Epidemiology

In the United States, BTC is rare and accounts for approximately 4% of all gastrointestinal malignancies, with an estimated 6000 to 7000 cases of carcinoma of the gallbladder and 3000 to 4000 cases of carcinoma of the bile duct diagnosed annually.4 Among women, there is a 26-fold variation in BTC mortality worldwide, ranging from 0.8 deaths per 100,000 in South Africa to 21.2 per 100,000 in Chile.1,5 Interestingly, for American Indians in New Mexico, gallbladder cancer mortality rates (8.9 per 100,000) surpass those for breast and pancreatic cancers.6 The incidence of anatomical cholangiocarcinoma subtypes also varies regionally, reflecting disparities in genetic and environmental predisposing factors.2,7 In a large, single-center study in the United States, intrahepatic cholangiocarcinoma accounted for less than 10% of cases, perihilar accounted for 50%, and distal accounted for the remaining 40%.8 Importantly, intrahepatic cholangiocarcinoma is the second most common primary malignancy of the liver, and its incidence seems to be rising in many western countries. In the United States, there has been an estimated 128% rise over the past 40 years.4,9

BTC is associated with high mortality rates.10 Median overall survival (OS) for cholangiocarcinoma is 20 to 28 months and 5-year survival is around 25%.10 Most cholangiocarcinomas are diagnosed at advanced stages with unresectable tumors.10 Furthermore, outcomes following resection with curative intent are poor—median disease-free survival (DFS) of 12 to 36 months has been reported.11,12 Patients with intrahepatic disease have a better prognosis when compared with patients who have extrahepatic tumors.12 Gallbladder cancer, likewise, carries a poor overall prognosis; median OS is 32 months and 5-year survival is as low as 13%.6

Risk factors for BTC include intrinsic and extrinsic elements.6 Incidence of BTC increases with age, and diagnosis typically occurs in the sixth to eighth decade of life.5,6,13 In contrast to gallbladder cancer, the incidence of cholangiocarcinoma is slightly higher in men.9 Obesity, diabetes, and consumption of sweetened drinks also increase the risk for BTC.14–16 Cholelithiasis is the most prevalent risk factor for gallbladder cancer, and the risk is greater for larger stones.5 Around 1 in 5 patients with porcelain gallbladder will develop gallbladder carcinoma.17 Primary sclerosing cholangitis (PSC), chronic calculi of the bile duct, choledochal cysts, cirrhosis, hepatitis C, and liver fluke infections are well established risk factors for cholangiocarcinoma.7,12,18 PSC is one of the best described entities among these predisposing conditions. Lifetime prevalence of cholangiocarcinoma among patients with PSC ranges from 5% to 10%.18,19 These patients also present at a younger age; in one series, the median age at diagnosis for BTC arising from PSC was 39 years.18 It is important to recognize, however, that in most patients diagnosed with cholangiocarcinoma, no predisposing factors are identified.8

Diagnosis

Clinical Presentation

Clinical presentation of BTC depends upon anatomic location.20 Patients with early invasive gallbladder cancer are most often asymptomatic.21 When symptoms occur, they may be nonspecific and mimic cholelithiasis.21 The most common clinical presentations include jaundice, weight loss, and abdominal pain.21 Prior to widespread availability of imaging studies, the preoperative diagnosis rate for gallbladder cancer was as low as 10%.22 However, the accuracy of computed tomography (CT) has changed this scenario, with sensitivity ranging from 73% to 87% and specificity from 88% to 100%.21 As a result of its silent clinical character, cholangiocarcinoma is frequently difficult to diagnose.23 Perihilar and distal cholangiocarcinoma characteristically present with signs of biliary obstruction, and imaging and laboratory data can corroborate the presence of cholestasis.24 On examination, patients with extrahepatic cholangiocarcinoma may present with jaundice, hepatomegaly, and a palpable right upper quadrant mass.25 A palpable gallbladder (Courvoisier sign) can also be present.25 Intrahepatic cholangiocarcinoma presents differently, and patients are less likely to be jaundiced.23 Typical clinical features are nonspecific and include dull right upper quadrant pain, weight loss, and an elevated alkaline phosphatase level.23 Alternatively, asymptomatic patients can present with incidentally detected lesions, when imaging is obtained as part of the workup for other causes or during screening for hepatocellular carcinoma in patients with viral hepatitis or cirrhosis.23,26 Uncommonly, BTC patients present because of signs or symptoms related to metastatic disease or evidence of metastatic disease on imaging.

 

 

Pathology and Grading

The majority of BTCs are adenocarcinomas, corresponding to 90% of cholangiocarcinomas and 99% of gallbladder cancers.27,28 They are graded as well, moderately, or poorly differentiated.2 Adenosquamous and squamous cell carcinoma are responsible for most of the remaining cases.2,29 Cholangiocarcinomas are divided into 3 types, defined by the Liver Cancer Study Group of Japan: (1) mass-forming, (2) periductal-infiltrating, and (3) intraductal-growing.30,31 Mass-forming intrahepatic cholangiocarcinomas are characterized morphologically by a homogeneous gray-yellow mass with frequent satellite nodules and irregular but well-defined margins.17,30 Central necrosis and fibrosis are also common.30 In the periductal-infiltrating type, tumor typically grows along the bile duct wall without mass formation, resulting in concentric mural thickening and proximal biliary dilation.30 Intraductal-growing papillary cholangiocarcinoma is characterized by the presence of intraluminal papillary or tubular polypoid tumors of the intra- or extrahepatic bile ducts, with partial obstruction and proximal biliary dilation.30

Cholangiocarcinoma

Case Presentation

A previously healthy 59-year-old man presents to his primary care physician with a 3-month history of dull right upper quadrant pain associated with weight loss. The patient is markedly cachectic and abdominal examination reveals upper quadrant tenderness. Laboratory exams are significant for elevated alkaline phosphatase (500 U/L; reference range 45–115 U/L), cancer antigen 19-9 (CA 19-9, 73 U/mL; reference range ≤ 37 U/mL), and carcinoembryonic antigen (CEA , 20 ng/mL; reference range for nonsmokers ≤ 3.0 ng/mL). Aspartate aminotransferase, alanine aminotransferase, total bilirubin, and coagulation studies are within normal range. Ultrasound demonstrates a homogeneous mass with irregular borders in the right lobe of the liver. Triphasic contrast-enhanced CT scan demonstrates a tumor with ragged rim enhancement at the periphery, and portal venous phase shows gradual centripetal enhancement of the tumor with capsular retraction. No abdominal lymph nodes or extrahepatic tumors are noted (Figure 1, Image A).

  • What are the next diagnostic steps?

The most critical differential diagnosis of solid liver mass in patients without cirrhosis is cholangiocarcinoma and metastases from another primary site.32 Alternatively, when an intrahepatic lesion is noted on an imaging study in the setting of cirrhosis, the next diagnostic step is differentiation between cholangiocarcinoma and hepatocellular carcinoma (HCC).32 Triphasic contrast-enhanced CT and dynamic magnetic resonance imaging (MRI) are key diagnostic procedures.32,33 In the appropriate setting, classical imaging features in the arterial phase with washout in portal venous or delayed phase can be diagnostic of HCC and may obviate the need for a biopsy (Figure 2).

Typical radiographic features of cholangiocarcinoma include a hypodense hepatic lesion that can be either well-defined or infiltrative and is frequently associated with biliary dilatation (Figure 1, Image A).33 The dense fibrotic nature of the tumor may cause capsular retraction, which is seen in up to 20% of cases.17 This finding is highly suggestive of cholangiocarcinoma and is rarely present in HCC.33 Following contrast administration, there is peripheral (rim) enhancement throughout both arterial and venous phases.32–34 However, these classic features were present in only 70% of cases in one study.35 Although intrahepatic cholangiocarcinomas are most commonly hypovascular, small mass-forming intrahepatic cholangiocarcinomas can often be arterially hyperenhancing and mimic HCC.33 Tumor enhancement on delayed CT imaging has been correlated with survival. Asayama et al demonstrated that tumors that exhibited delayed enhancement on CT in more than two-thirds of their volume were associated with a worse prognosis.36

Patients without cirrhosis who present with a localized lesion of the liver should undergo extensive evaluation for a primary cancer site.37 CT of the chest, abdomen, and pelvis with contrast should be obtained.37 Additionally, mammogram and endoscopic evaluation with esophagogastroduodenoscopy (EGD) and colonoscopy should be included in the work-up.37

Preoperative tumor markers are also included in the work-up. All patients with a solid liver lesion should have serum alpha-fetoprotein (AFP) levels checked. AFP is a serum glycoprotein recognized as a marker for HCC and is reported to detect preclinical HCC.38 However, serum concentrations are normal in up to 40% of small HCCs.38 Although no specific marker for cholangiocarcinoma has yet been identified, the presence of certain tumor markers in the serum of patients may be of diagnostic value, especially in patients with PSC. CA 19-9 and CEA are the best studied. Elevated levels of CA 19-9 prior to treatment are associated with a poorer prognosis, and CA 19-9 concentrations greater than 1000 U/mL are consistent with advanced disease.39,40 One large series evaluated the diagnostic value of serum CEA levels in 333 patients with PSC, 13% of whom were diagnosed with cholangiocarcinoma.34 A serum CEA level greater than 5.2 ng/mL had a sensitivity of 68.0% and specificity of 81.5%.38

If a biopsy is obtained, appropriate immunohistochemistry (IHC) can facilitate the diagnosis. BTC is strongly positive for CK-7 and CK-19.41 CK-7 positivity is not specific and is also common among metastatic cancers of the lung and breast; therefore, in some cases cholangiocarcinoma may be a diagnosis of exclusion. Immunostaining for monoclonal CEA is diffusely positive in up to 75% of cases.41 An IHC panel consisting of Hep Par-1, arginase-1, monoclonal CEA, CK-7, CK-20, TTF-1, MOC-31, and CDX-2 has been proposed to optimize the differential diagnosis of HCC, metastatic adenocarcinoma, and cholangiocarcinoma.41

 

 

Case Continued

CT of the chest, abdomen, and pelvis reveals no concerns for metastasis and no evidence of primary cancer elsewhere. EGD and colonoscopy are clear. AFP levels are within normal limits (2 ng/mL). Biopsy is performed and demonstrates adenocarcinoma. IHC studies demonstrate cells positive for monoclonal CEA, CK-7, CK-19, and MOC-31, and negative for Napsin A, TTF-1, and CK-20.

  • How is cholangiocarcinoma staged and classified?

The purpose of the staging system is to provide information on prognosis and guidance for therapy. Prognostic factors and the therapeutic approaches for BTC differ depending upon their location in the biliary tree. Accordingly, TNM classification systems for intrahepatic, hilar, and distal cholangiocarcinoma and gallbladder cancer have been separated (Table 1 and Table 2).23

For all the subtypes, T stage is mainly dependent upon invasion of adjacent structures rather than size. For perihilar tumors, N category has been reclassified in the newest version of the American Joint Committee on Cancer/Union for International Cancer Control (AJCC/UICC) staging system based upon the number of involved lymph nodes rather than location.

The Bismuth-Corlette classification is used to further classify perihilar cholangiocarcinoma according to patterns of hepatic duct involvement. Type I tumors are located below the confluence of the left and right hepatic ducts.42 Type II reach the confluence of the hepatic ducts.42 Type III occlude the common hepatic duct and either the right or left hepatic duct (IIIa and IIIb, respectively).42 Finally, type IV are multicentric, or involve the confluence and both the right and left hepatic ducts.42 Tumors that involve the common hepatic duct bifurcation are named Klatskin tumors.42

  • What is the first-line treatment for localized cholangiocarcinomas?

Surgical resection is the only potentially curative treatment for localized cholangiocarcinoma, although fewer than 20% of patients are suitable for curative treatment, due to the presence of advanced disease at diagnosis.43,44 Available evidence supports the recommendation that resection with negative margins, regardless of extent, should be the goal of therapy for patients with potentially resectable disease.44 Extensive hepatic resections are often necessary to achieve clear margins since the majority of patients present with large masses. Substantial evidence corroborates that R0 resection is associated with better survival, whereas the benefit of wide compared to narrow (< 5–10 mm) margins is unclear.45 A recent analysis of 96 patients suggests that the proximal resection margin has more prognostic implications than distal margins.45

Surgical options and resectability criteria depend upon tumor location. Extent of tumor in the bile duct is one of the most important factors that determine resectability.17 Although multifocal liver tumors (including satellite lesions), lymph node metastases to the porta hepatis, and distant metastases are considered relative contraindications to surgery, surgical approaches can be considered in selected patients.43 Patient selection for surgery is facilitated by careful preoperative staging, which may include laparoscopy. Laparoscopic staging prior to resection may prevent unnecessary laparotomy in 30% to 45% of patients.42,46

  • Is there a role for adjuvant treatment?

Recurrence following complete resection is a primary limitation for cure in BTC, which provides a rationale for the use of adjuvant therapy.47,48 In a sample of 79 patients with extrahepatic cholangiocarcinoma who underwent curative resection, the cumulative recurrence rate after 4 years was 56%.47 Initial recurrence at a distant site occurs in 40% to 50% of patients.48

Lymphovascular and perineural invasion, lymph node metastasis, and tumor size ≥ 5 cm have been reported as independent predictors of recurrence and mortality following resection.49 A 2017 meta-analysis which included 30 studies involving more than 22,499 patients reported a 41% reduction in the risk of death with adjuvant chemotherapy, which translated to a mean OS benefit of 4 months in an unselected population.49 Moreover, this study revealed inferior OS in patients given adjuvant radiation therapy (RT) in combination with chemotherapy.49 These results are in line with the previous meta-analysis by Horgan et al, which demonstrated that adjuvant RT seems to benefit only patients with R1 resections, with a possible detrimental effect in R0 disease.50 Therefore, adjuvant chemoradiation cannot be viewed as a standard practice following R0 resection, and should be reserved for those patients with positive margins (R1/ 2) to reduce local progression.

In the phase 3 BILCAP trial presented at ASCO 2017, 447 patients with completely resected cholangiocarcinoma or gallbladder cancer with adequate biliary drainage and Eastern Cooperative Oncology Group (ECOG) performance score ≤ 2 were randomly assigned to observation or capecitabine (1250 mg/m2 twice daily for days 1–14 every 21 days for 8 cycles).51 Surgical treatment achieved R0 resection in 62% of patients and 46% were node-negative. Median OS was 51 months for the capecitabine group and 36 months for the control arm (hazard ratio [HR] 0.80, 95% CI 0.63 to 1.04, P = 0.097). Analyses with adjustment for nodal status, grade of disease, and gender indicated a HR of 0.71 (P < 0.01). Median DFS was 25 months versus 18 months favoring the capecitabine group, and rates of grade 3 or 4 toxicity were less than anticipated. Following the results of this trial, adjuvant capecitabine should become the new standard of care.

 

 

  • What is the treatment for locally advanced cholangiocarcinoma?

The optimal approach to patients with locally advanced unresectable cholangiocarcinoma has not been established. The prognosis for patients with either locally unresectable or locally recurrent disease is typically measured in months. Goals of palliative therapy are relief of symptoms and improvement in quality of life, and there is no role for surgical debulking.

Liver transplantation is a potentially curative option for selected patients with hilar or intrahepatic cholangiocarcinoma. Patients with lymph node-negative, non-disseminated, locally advanced hilar cholangiocarcinomas have 5-year survival rates ranging from 25% to 42% following transplantation.52 Retrospective data suggests that neoadjuvant chemoradiation followed by liver transplantation is highly effective for selected patients with hilar cholangiocarcinoma.52 However, these results require confirmation from prospective clinical evidence. It is important to recognize that liver transplantation plays no role in the management of distal cholangiocarcinoma or gallbladder cancer.

Rarely, patients with borderline resectable intrahepatic cholangiocarcinoma will have a sufficient response to chemotherapy to permit later resection, and, in such cases, starting with chemotherapy and then restaging to evaluate resectability is appropriate.54 A single-center, retrospective analysis including 186 patients by Le Roy et al evaluated survival in patients with locally advanced, unresectable intrahepatic cholangiocarcinoma who received primary chemotherapy, followed by surgery in those with secondary resectability.54 After a median of 6 cycles of chemotherapy, 53% of patients achieved resectability and underwent surgery with curative intent. These patients had similar short- and long-term results compared to patients with initially resectable intrahepatic cholangiocarcinoma who had surgery alone, with median OS reaching 24 months.54

Ablative radiotherapy is an additional option for localized inoperable intrahepatic cholangiocarcinoma. Tao and colleagues evaluated 79 consecutive patients with inoperable intrahepatic cholangiocarcinoma treated with definitive RT.55 Median tumor size was 7.9 cm and 89% of patients received chemotherapy before RT. Median OS was 30 months and 3-year OS was 44%. Radiation dose was the single most important prognostic factor, and higher doses correlated with improved local control and OS. A biologic equivalent dose (BED) greater than 80.5 Gy was identified as an ablative dose of RT for large intrahepatic cholangiocarcinomas. The 3-year OS for patients receiving BED greater than 80.5 Gy was 73% versus 38% for those receiving lower doses.

Case Continued

The patient is deemed to have resectable disease and undergoes surgical resection followed by adjuvant capecitabine for 8 cycles. Unfortunately, after 1 year, follow-up imaging identifies bilateral enlarging lung nodules. Biopsy is performed and confirms metastatic cholangiocarcinoma.

  • What is the treatment for metastatic BTC?

The prognosis of patients with advanced BTC is poor and OS for those undergoing supportive care alone is short. A benefit of chemotherapy over best supportive care for cholangiocarcinoma was demonstrated in an early phase 3 trial that randomly assigned 90 patients with advanced pancreatic or biliary cancer (37 with bile duct cancer) to receive either fluorouracil (FU) -based systemic chemotherapy or best supportive care. Results showed that chemotherapy significantly improved OS (6 months versus 2.5 months).56 Chemotherapy is also beneficial for patients with unresectable gallbladder cancer. In a single-center randomized study including 81 patients with unresectable gallbladder cancer, gemcitabine and oxaliplatin (GEMOX) improved progression-free survival (PFS) and OS compared to best supportive care.57 Treatment for metastatic cholangiocarcinoma and gallbladder cancer follows the same algorithm.

In 2010, cisplatin plus gemcitabine was established as a reference regimen for first-line therapy by the ABC-02 study, in which 410 patients with locally advanced or metastatic bile duct, gallbladder, or ampullary cancer were randomly assigned to 6 courses of cisplatin (25 mg/m2) plus gemcitabine (1000 mg/m2 on days 1 and 8, every 21 days) or gemcitabine alone (1000 mg/m2 days 1, 8, 15, every 28 days).58 OS was significantly greater with combination therapy (11.7 versus 8.1 months), and PFS also favored the combination arm (8 versus 5 months). Toxicity was comparable in both groups, with the exception of significantly higher rates of grade 3 or 4 neutropenia with gemcitabine plus cisplatin (25% versus 17%), and higher rates of grade 3 or 4 abnormal liver function with gemcitabine alone (27% versus 17%). Most quality-of-life scales showed a trend favoring combined therapy.58 A smaller, identically designed Japanese phase 3 randomized trial achieved similar results, demonstrating greater OS with cisplatin plus gemcitabine compared to gemcitabine alone (11.2 versus 7.7 months).59

The gemcitabine plus cisplatin combination has not been directly compared with other gemcitabine combinations in phase 3 trials. A pooled analysis of 104 trials of a variety of chemotherapy regimens in advanced biliary cancer concluded that the gemcitabine plus cisplatin regimen offered the highest rates of objective response and tumor control compared with either gemcitabine-free or cisplatin-free regimens.60 However, this did not translate into significant benefit in terms of either time to tumor progression or median OS. It is important to note that this analysis did not include results of the subsequent ABC-02 trial.

There is no standard treatment for patients with cholangiocarcinoma for whom first-line gemcitabine-based therapy fails. There are no completed prospective phase 3 trials supporting the use of second-line chemotherapy after failure of first-line chemotherapy in BTC, and the selection of candidates for second-line therapy as well as the optimal regimen are not established.61 The ongoing phase 2 multicenter ABC-06 trial is evaluating oxaliplatin plus short-term infusional FU and leucovorin (FOLFOX) versus best supportive care for second-line therapy. In a systematic review including 23 studies (14 phase 2 clinical trials and 9 retrospective studies) with 761 patients with BTC, the median OS was 7.2 months.

The optimal selection of candidates for second-line chemotherapy is not established. Two independent studies suggest that patients who have a good performance status (0 or 1), disease control with the first-line chemotherapy, low CA 19-9 level, and possibly previous surgery on their primary tumor, have the longest survival with second-line chemotherapy. However, whether these characteristics predict for chemotherapy responsiveness or more favorable biologic behavior is not clear.62,63 No particular regimen has proved superior to any other, and the choice of second-line regimen remains empiric.

For patients with adequate performance status, examples of other conventional chemotherapy regimens with demonstrated activity that could be considered for second-line therapy include: FOLFOX or capecitabine, gemcitabine plus capecitabine, capecitabine plus cisplatin, or irinotecan plus short-term infusional FU and leucovorin (FOLFIRI) with or without bevacizumab.64 For selected patients, second-line molecularly targeted therapy using erlotinib plus bevacizumab may be considered. However, this regimen is very costly.64 Examples of other regimens with demonstrated activity in phase 2 trials include GEMOX, gemcitabine plus fluoropyrimidine, and fluoropyrimidine plus oxaliplatin or cisplatin.64

There is promising data from studies of targeted therapy for specific molecular subgroups. A recent phase 2 trial evaluated the activity of BGJ398, an orally bioavailable, selective, ATP-competitive pan inhibitor of human fibroblast growth factor receptor (FGFR) kinase, in patients with FGFR-altered advanced cholangiocarcinoma.65 The overall response rate was 14.8% (18.8% FGFR2 fusions only) and disease control rate was 75.4% (83.3% FGFR2 fusions only). All responsive tumors contained FGFR2 fusions. Adverse events were manageable, and grade 3 or 4 treatment-related adverse events occurred in 25 patients (41%). Those included hyperphosphatemia, stomatitis, and palmar-plantar erythrodysesthesia. Javle and colleagues also identified HER2/neu blockade as a promising treatment strategy for gallbladder cancer patients with this gene amplification.66 This retrospective analysis included 9 patients with gallbladder cancer and 5 patients with cholangiocarcinoma who received HER2/neu-directed therapy (trastuzumab, lapatinib, or pertuzumab). In the gallbladder cancer group, HER2/neu gene amplification or overexpression was detected in 8 cases. These patients experienced disease stability (n = 3), partial response (n = 4), or complete response (n = 1) with HER2/neu–directed therapy. Median duration of response was 40 weeks. The cholangiocarcinoma cases treated in this series had no radiological responses despite HER2/neu mutations or amplification.

 

 

Gallbladder Cancer

Case Presentation

A 57-year-old woman from Chile presents with a 3-week history of progressive right upper quadrant abdominal pain. She denies nausea, vomiting, dysphagia, odynophagia, alterations in bowel habits, fever, or jaundice. Her past medical history is significant for obesity and hypertension. She has no history of smoking, alcohol, or illicit drug use. Laboratory studies show marked leukocytosis (23,800/µL) with neutrophilia (91%). Liver function test results are within normal limits. Ultrasound of the abdomen reveals gallbladder wall thickening and cholelithiasis.

The patient undergoes an uneventful laparoscopic cholecystectomy and is discharged from the hospital after 48 hours. Pathology report reveals a moderately differentiated adenocarcinoma of the gallbladder invading the perimuscular connective tissue (T2). No lymph nodes are identified in the specimen.

  • What is the appropriate surgical management of gallbladder cancer?

Gallbladder cancer can be diagnosed preoperatively or can be found incidentally by intraoperative or pathological findings. In one large series, gallbladder cancer was incidentally found during 0.25% of laparoscopic cholecystectomies.67

For patients who are diagnosed with previously unsuspected gallbladder cancer by pathology findings, the extent of tumor invasion (T stage) indicates the need for re-resection (Figure 3).64

Surgical exploration and re-resection are recommended if disease is stage T1b (involving the muscular layer) or higher (Table 2).64,68 In these patients, re-resection is associated with significantly improved OS.68 Patients found to have incidental T1a tumors with negative margins are generally felt to be curable with simple cholecystectomy, and re-resection for T1a tumors does not appear to provide an OS benefit.69,70 The majority of patients diagnosed under these circumstances have T2 or higher disease, and will ultimately require additional surgical exploration.71 A German series that analyzed 439 cases of incidentally diagnosed gallbladder cancer demonstrated that positive lymph nodes were found in 21% and 44% of the re-resected patients with T2 and T3 tumors, respectively.71 There is retrospective data suggesting that the optimal timing of the reoperation is between 4 and 8 weeks following the initial cholecystectomy.72 This interval is believed to be ideal, as it allows for reduced inflammation and does not permit too much time for disease dissemination.72

Alternatively, when gallbladder cancer is documented or suspected preoperatively, adequate imaging is important to identify patients with absolute contraindications to resection. Contraindications to surgery include metastasis, extensive involvement of the hepatoduodenal ligament, encasement of major vessels, and involvement of celiac, peripancreatic, periduodenal, or superior mesenteric nodes.72 Notwithstanding, retrospective series suggest individual patients may benefit, and surgical indications in advanced disease should be determined on an individual basis.73 Staging imaging should be obtained using multiphasic contrast-enhanced CT or MRI of the chest, abdomen, and pelvis. PET-scan can be used in selected cases where metastatic disease is suspected.64 Laparoscopic diagnostic staging should be considered prior to resection.64 This procedure can identify previously unknown contraindications to tumor resection in as much as 23% of patients, and the yield is significantly higher in locally advanced tumors.73

Patients with a diagnosis of potentially resectable, localized gallbladder cancer should be offered definitive surgery. Extended cholecystectomy is recommended for patients stage T2 or above. This procedure involves wedge resection of the gallbladder bed or a segmentectomy IVb/V and lymph node dissection, which should include the cystic duct, common bile duct, posterior superior pancreaticoduodenal lymph nodes, and those around the hepatoduodenal ligament.72 Bile duct excision should be performed if there is malignant involvement.64

Conclusion

BTCs are anatomically and clinically heterogeneous tumors. Prognostic factors and therapeutic approaches for BTCs differ depending upon their location in the biliary tree and, accordingly, TNM classification systems for intrahepatic, hilar, and distal cholangiocarcinoma and gallbladder cancer have been separated. Surgical resection is the only potentially curative treatment for localized BTC. However, recurrence following complete resection is a primary limitation for cure, which provides a rationale for the use of adjuvant therapy. The prognosis of patients with advanced BTC is poor and OS for those undergoing supportive care alone is short. Multiple randomized clinical trials have demonstrated a benefit of chemotherapy for metastatic disease. For patients with adequate performance status, second-line therapy can be considered, and data from studies that evaluated targeted therapy for specific molecular subgroups is promising.

References

1. Goldstein D, Lemech C, Valle J. New molecular and immunotherapeutic approaches in biliary cancer. ESMO Open 2017;2(Suppl 1):e000152.

2. Rizvi S, Khan SA, Hallemeier CL, et al. Cholangiocarcinoma - evolving concepts and therapeutic strategies. Nat Rev Clin Oncol 2017 Oct 10. doi: 10.1038/nrclinonc.2017.157.

3. Hezel AF, Zhu AX. Systemic therapy for biliary tract cancers. Oncologist 2008;13:415–23.

4. U.S. Cancer Statistics Working Group. United States Cancer Statistics: 1999-2014 Incidence and Mortality Web-based Report. Atlanta: U.S. Department of Health and Human Services, Centers for Disease Control and Prevention and National Cancer Institute; 2017.

5. Torre LA, Siegel RL, Islami F, et al. Worldwide burden of and trends in mortality from gallbladder and other biliary tract cancers. Clin Gastroenterol Hepatol 2017 Aug 18. doi: 10.1016/j.cgh.2017.08.017.

6. Lau CSM, Zywot A, Mahendraraj K, Chamberlain CS. Gallbladder carcinoma in the United States: a population based clinical outcomes study involving 22,343 patients from the Surveillance, Epidemiology, and End Result Database (1973–2013). HPB Surg 2017;2017:1532835. doi:10.1155/2017/1532835.

7. Hughes T, O’Connor T, Techasen A, et al. Opisthorchiasis and cholangiocarcinoma in Southeast Asia: an unresolved problem. Int J Gen Med 2017;10:227–37.

8. DeOliveira ML, Cunningham SC, Cameron JL, et al. Cholangiocarcinoma: thirty-one-year experience with 564 patients at a single institution. Ann Surg 2007;245:755–62.

9. Saha SK, Zhu AX, Fuchs CS, Brooks GA. Forty-year trends in cholangiocarcinoma incidence in the U.S.: intrahepatic disease on the rise. Oncologist 2016;21:594–9.

10. Yao KJ, Jabbour S, Parekh N, et al. Increasing mortality in the United States from cholangiocarcinoma: an analysis of the National Center for Health Statistics Database. BMC Gastroenterol 2016;16:117.

11. Choi SB, Kim KS, Choi JY, et al. The prognosis and survival outcome of intrahepatic cholangiocarcinoma following surgical resection: association of lymph node metastasis and lymph node dissection with survival. Ann Surg Oncol 2009;16:3048–56.

12. Endo I, Gonen M, Yopp AC, et al. Intrahepatic cholangiocarcinoma: rising frequency, improved survival, and determinants of outcome after resection. Ann Surg 2008;248:84–96.

13. Duffy A, Capanu M, Abou-Alfa GK, et al. Gallbladder cancer (GBC): 10-year experience at Memorial Sloan-Kettering Cancer Centre (MSKCC). J Surg Oncol 2008;98:485–9.

14. Lauby-Secretan B, Scoccianti C, Loomis D, et al. Body fatness and cancer — viewpoint of the IARC Working Group. N Engl J Med 2016;375:794–8.

15. Chen J, Han Y, Xu C, et al. Effect of type 2 diabetes mellitus on the risk for hepatocellular carcinoma in chronic liver diseases. Eur J Cancer Prev 2015;24:89–99.

16. Larsson SC, Giovannucci EL, Wolk A. Sweetened beverage consumption and risk of biliary tract and gallbladder cancer in a prospective study. J Natl Cancer Inst 2016;108: doi: 10.1093/jnci/djw125.

17. Gore RM. Biliary tract neoplasms: diagnosis and staging. Cancer Imaging 2007;7(Special Issue A):S15–23.

18. Broome U, Olsson R, Lööf L, et al. Natural history and prognostic factors in 305 Swedish patients with primary sclerosing cholangitis. Gut 1996;38:610–5.

19. Burak K, Angulo P, Pasha T, et al. Incidence and risk factors for cholangiocarcinoma in primary sclerosing cholangitis. Am J Gastroenterol 2004;99:523–6.

20. Rodrigues J, Diehl DL. Cholangiocarcinoma: clinical manifestations and diagnosis. Tech Gastrointest Endosc 2016;18:75–82.

21. Mitchell CH, Johnson PT, Fishman EK, et al. Features suggestive of gallbladder malignancy. J Comput Assist Tomogr 2014;38:235–41.

22. Beltz WR, Condon RE. Primary carcinoma of the gallbladder. Ann Surg 1974;180:180–4.

23. Blechacz B, Komuta M, Roskams T, Gores GJ. Clinical diagnosis and staging of cholangiocarcinoma. Nat Rev Gastroenterol Hepatol 2011;8:512–22.

24. Patel T. Cholangiocarcinoma—controversies and challenges. Nat Rev Gastroenterol Hepatol 2011;8:189–200.

25. Nakeeb A, Pitt HA, Sohn TA, et al. Cholangiocarcinoma. A spectrum of intrahepatic, perihilar, and distal tumors. Ann Surg 1996;224:463–73.

26. Bartella I, Dufour JF. Clinical diagnosis and staging of intrahepatic cholangiocarcinoma. J Gastrointestin Liver Dis 2015;24:481-9.

27. Yamaguchi K, Enjoji M. Carcinoma of the gallbladder: a clinicopathology of 103 patients and a newly proposed staging. Cancer 1988;62:1425–32.

28. Esposito I, Schirmacher P. Pathological aspects of cholangiocarcinoma. HPB. 2008;10:83–6.

29. Silva VWK, Askan G, Daniel TD, et al. Biliary carcinomas: pathology and the role of DNA mismatch repair deficiency. Chin Clin Oncol 2016;5:62.

30. Chung YE, Kim MJ, Park YN, et al. Varying appearances of cholangiocarcinoma: radiologic-pathologic correlation. Radiographics 2009;29:683–700.

31. Yamasaki S. Intrahepatic cholangiocarcinoma: macroscopic type and stage classification. J Hepatobiliary Pancreat Surg 2003;10:288–91.

32. Rao PN. Nodule in liver: investigations, differential diagnosis and follow-up. J Clin Exp Hepatol 2014;4(Suppl 3):S57–62.

33. Kim TK, Lee E, Jang HJ. Imaging findings of mimickers of hepatocellular carcinoma. Clin Mol Hepatol 2015;21:326–43.

34. Hennedige TP, Neo WT, Venkatesh SK. Imaging of malignancies of the biliary tract- an update. Cancer Imaging 2014;14:14.

35. Kim SH, Lee CH, Kim BH, et al. Typical and atypical imaging findings of intrahepatic cholangiocarcinoma using gadolinium ethoxybenzyl diethylenetriamine pentaacetic acid-enhanced magnetic resonance imaging. J Comput Assist Tomogr 2012;36:704–9.

36. Asayama Y, Yoshimitsu K, Irie H, et al. Delayed-phase dynamic CT enhancement as a prognostic factor for mass-forming intrahepatic cholangiocarcinoma. Radiology 2006;238:150–5.

37. National Comprehensive Cancer Network. Cancer of unknown primary. www.nccn.org/professionals/physician_gls/pdf/bone.pdf. Accessed 1 Dec 2017.

38. Kefeli A, Basyigit S, Yeniova AO. Diagnosis of hepatocellular carcinoma. In: Abdeldayem HM, ed. Updates in liver cancer. London: InTech; 2017.

39. Bergquist JR, Ivanics T, Storlie CB, et al. Implications of CA19-9 elevation for survival, staging, and treatment sequencing in intrahepatic cholangiocarcinoma: A national cohort analysis. J Surg Oncol 2016;114:475–82.

40. Chung YJ, Choi DW, Choi SH, et al. Prognostic factors following surgical resection of distal bile duct cancer. J Korean Surg Soc 2013;85:212–8.

41. Lau SK, Prakash S, Geller SA, Alsabeh R. Comparative immunohistochemical profile of hepatocellular carcinoma, cholangiocarcinoma, and metastatic adenocarcinoma. Hum Pathol 2002;33:1175–81.

42. Paul A, Kaiser GM, Molmenti EP, et al. Klatskin tumors and the accuracy of the Bismuth-Corlette classification. Am Surg 2011;77:1695–9.

43. Cannavale A, Santoni M, Gazzetti M, et al. Updated management of malignant biliary tract tumors: an illustrative review. J Vasc Interv Radiol 2016;27:1056–69.

44. Matsuo K, Rocha FG, Ito K, et al. The Blumgart preoperative staging system for hilar cholangiocarcinoma: analysis of resectability and outcomes in 380 patients. J Am Coll Surg 2012;215:343–55.

45. Yoo T, Park SJ, Han SS, et al. Proximal resection margins: more prognostic than distal resection margins in patients undergoing hilar cholangiocarcinoma resection. Cancer Res Treat 2017 Nov 16; doi.org/10.4143/crt.2017.320.

46. Joseph S, Connor S, Garden OJ. Staging laparoscopy for cholangiocarcinoma. HPB 2008;10:116–9.

47. Jarnagin WR, Ruo L, Little SA, et al. Patterns of initial disease recurrence after resection of gallbladder carcinoma and hilar cholangiocarcinoma: implications for adjuvant therapeutic strategies. Cancer 2003;98:1689–700.

48. Kobayashi A, Miwa S, Nakata T, Miyagawa S. Disease recurrence patterns after R0 resection of hilar cholangiocarcinoma. Br J Surg 2010;97:56–64.

49. Ghidini M, Tomasello G, Botticelli A, et al. Adjuvant chemotherapy for resected biliary tract cancers: a systematic review and meta-analysis. HPB 2017;19:741–8.

50. Horgan AM, Amir E, Walter T, Knox JJ. Adjuvant therapy in the treatment of biliary tract cancer: a systematic review and meta-analysis. J Clin Oncol 2012;30:1934–40.

51. Primrose JN, Fox R, Palmer DH, et al. Adjuvant capecitabine for biliary tract cancer: the BILCAP randomized study [abstract]. J Clin Oncol 2017 35:15_suppl:4006-4006. 

52. Darwish Murad S, Kim WR, Darnois DM, et al. Efficacy of neoadjuvant chemoradiation followed by liver transplantation for perihilar cholangiocarcinoma at 12 US centers. Gastroenterology 2012;143:88–98.

53. Sapisochin G, Facciuto M, Rubbia-Brandt L, et al. Liver transplantation for “very early” intrahepatic cholangiocarcinoma: International retrospective study supporting a prospective assessment. Hepatology 2016;64:1178–88.

54. Le Roy B, Gelli M, Pittau G, et al. Neoadjuvant chemotherapy for initially unresectable intrahepatic cholangiocarcinoma. Br J Surg 2017 Aug 31. doi: 10.1002/bjs.10641.

55. Tao R, Krishnan S, Bhosale PR, et al. Ablative radiotherapy doses lead to a substantial prolongation of survival in patients with inoperable intrahepatic cholangiocarcinoma: a retrospective dose response analysis. J Clin Oncol 2016;34:219–26.

56. Glimelius B, Hoffman K, SjÓdén PO, et al. 555 Palliative chemotherapy improves survival and quality of life in advanced pancreatic and biliary cancer. Eur J Cancer 1995;31:S118.

57. Sharma A, Dwary AD, Mohanti BK, et al. Best supportive care compared with chemotherapy for unresectable gall bladder cancer: a randomized controlled study. J Clin Oncol 2010;28:4581–6.

58. Valle J, Wasan H, Palmer DH, et al. Cisplatin plus gemcitabine versus gemcitabine for biliary tract cancer. N Engl J Med 2010;362:1273–81.

59. Okusaka T, Nakachi K, Fukutomi A, et al. Gemcitabine alone or in combination with cisplatin in patients with biliary tract cancer: a comparative multicentre study in Japan. Br J Cancer 2010;103:469–74.

60. Eckel F, Schmid RM. Chemotherapy in advanced biliary tract carcinoma: a pooled analysis of clinical trials. Br J Cancer 2007;96:896–902.

61. Lamarca A, Hubner RA, David Ryder W, Valle JW. Second-line chemotherapy in advanced biliary cancer: a systematic review. Ann Oncol 2014;25:2328–38.

62. Brieau B, Dahan L, De Rycke Y, et al. Second-line chemotherapy for advanced biliary tract cancer after failure of the gemcitabine-platinum combination: A large multicenter study by the Association des Gastro-Entérologues Oncologues. Cancer 2015;121:3290–7.

63. Fornaro L, Cereda S, Aprile G, et al. Multivariate prognostic factors analysis for second-line chemotherapy in advanced biliary tract cancer. Br J Cancer 2014;110:2165–9.

64. National Comprehensive Cancer Network. Hepatobiliary cancer. www.nccn.org/professionals/physician_gls/pdf/hepatobiliary.pdf. Accessed 12 Nov 2017.

65. Javle M, Lowery M, Shroff RT, et al. Phase II study of BGJ398 in patients with FGFR-altered advanced cholangiocarcinoma. J Clin Oncol 2017 Nov 28;JCO2017755009.

66. Javle M, Churi C, Kang HC, et al. HER2/neu-directed therapy for biliary tract cancer. J Hematol Oncol 2015;8:58.

67. Konstantinidis IT, Deshpande V, Genevay M, et al. Trends in presentation and survival for gallbladder cancer during a period of more than 4 decades: a single-institution experience. Arch Surg 2009;144:441–47.

68. Singh S, Agarwal AK. Gallbladder cancer: the role of laparoscopy and radical resection. Ann Surg 2009;250:494–5.

69. Kapoor VK, Haribhakti SP. Extended cholecystectomy for carcinoma of the gall bladder. Trop Gastroenterol 1995;16:74–5.

70. Ethun CG, Postlewait LM, Le N, et al. Association of optimal time Interval to re-resection for incidental gallbladder cancer with overall survival: a multi-Institution analysis from the US extrahepatic biliary malignancy consortium. JAMA Surg 2017;152:143–9.

71. Goetze TO, Paolucci V. Benefits of reoperation of T2 and more advanced incidental gallbladder carcinoma: analysis of the German registry. Ann Surg 2008;247:104–8.

72. Nishio H, Nagino M, Ebata T, et al. Aggressive surgery for stage IV gallbladder carcinoma; what are the contraindications? J Hepatobiliary Pancreat Surg 2007;14:351–7.

73. Agarwal AK, Kalayarasan R, Javed A, et al. The role of staging laparoscopy in primary gallbladder cancer--an analysis of 409 patients: a prospective study to evaluate the role of staging laparoscopy in the management of gallbladder cancer. Ann Surg 2013;258:318–23.

Issue
Hospital Physician: Hematology/Oncology - 13(1)
Publications
Topics
Sections

Introduction

Biliary tract carcinoma (BTC) is the term for a heterogeneous group of rare gastrointestinal malignancies1 that includes both carcinoma arising from the gallbladder and cholangiocarcinoma, which refers to diverse aggressive epithelial cancers involving the intrahepatic, perihilar, and distal biliary tree.1–3 In this article, we review the epidemiology, clinical features, and diagnostic approach to BTC, with a focus on current evidence-based treatment strategies for localized, locally advanced, and metastatic BTC.

Epidemiology

In the United States, BTC is rare and accounts for approximately 4% of all gastrointestinal malignancies, with an estimated 6000 to 7000 cases of carcinoma of the gallbladder and 3000 to 4000 cases of carcinoma of the bile duct diagnosed annually.4 Among women, there is a 26-fold variation in BTC mortality worldwide, ranging from 0.8 deaths per 100,000 in South Africa to 21.2 per 100,000 in Chile.1,5 Interestingly, for American Indians in New Mexico, gallbladder cancer mortality rates (8.9 per 100,000) surpass those for breast and pancreatic cancers.6 The incidence of anatomical cholangiocarcinoma subtypes also varies regionally, reflecting disparities in genetic and environmental predisposing factors.2,7 In a large, single-center study in the United States, intrahepatic cholangiocarcinoma accounted for less than 10% of cases, perihilar accounted for 50%, and distal accounted for the remaining 40%.8 Importantly, intrahepatic cholangiocarcinoma is the second most common primary malignancy of the liver, and its incidence seems to be rising in many western countries. In the United States, there has been an estimated 128% rise over the past 40 years.4,9

BTC is associated with high mortality rates.10 Median overall survival (OS) for cholangiocarcinoma is 20 to 28 months and 5-year survival is around 25%.10 Most cholangiocarcinomas are diagnosed at advanced stages with unresectable tumors.10 Furthermore, outcomes following resection with curative intent are poor—median disease-free survival (DFS) of 12 to 36 months has been reported.11,12 Patients with intrahepatic disease have a better prognosis when compared with patients who have extrahepatic tumors.12 Gallbladder cancer, likewise, carries a poor overall prognosis; median OS is 32 months and 5-year survival is as low as 13%.6

Risk factors for BTC include intrinsic and extrinsic elements.6 Incidence of BTC increases with age, and diagnosis typically occurs in the sixth to eighth decade of life.5,6,13 In contrast to gallbladder cancer, the incidence of cholangiocarcinoma is slightly higher in men.9 Obesity, diabetes, and consumption of sweetened drinks also increase the risk for BTC.14–16 Cholelithiasis is the most prevalent risk factor for gallbladder cancer, and the risk is greater for larger stones.5 Around 1 in 5 patients with porcelain gallbladder will develop gallbladder carcinoma.17 Primary sclerosing cholangitis (PSC), chronic calculi of the bile duct, choledochal cysts, cirrhosis, hepatitis C, and liver fluke infections are well established risk factors for cholangiocarcinoma.7,12,18 PSC is one of the best described entities among these predisposing conditions. Lifetime prevalence of cholangiocarcinoma among patients with PSC ranges from 5% to 10%.18,19 These patients also present at a younger age; in one series, the median age at diagnosis for BTC arising from PSC was 39 years.18 It is important to recognize, however, that in most patients diagnosed with cholangiocarcinoma, no predisposing factors are identified.8

Diagnosis

Clinical Presentation

Clinical presentation of BTC depends upon anatomic location.20 Patients with early invasive gallbladder cancer are most often asymptomatic.21 When symptoms occur, they may be nonspecific and mimic cholelithiasis.21 The most common clinical presentations include jaundice, weight loss, and abdominal pain.21 Prior to widespread availability of imaging studies, the preoperative diagnosis rate for gallbladder cancer was as low as 10%.22 However, the accuracy of computed tomography (CT) has changed this scenario, with sensitivity ranging from 73% to 87% and specificity from 88% to 100%.21 As a result of its silent clinical character, cholangiocarcinoma is frequently difficult to diagnose.23 Perihilar and distal cholangiocarcinoma characteristically present with signs of biliary obstruction, and imaging and laboratory data can corroborate the presence of cholestasis.24 On examination, patients with extrahepatic cholangiocarcinoma may present with jaundice, hepatomegaly, and a palpable right upper quadrant mass.25 A palpable gallbladder (Courvoisier sign) can also be present.25 Intrahepatic cholangiocarcinoma presents differently, and patients are less likely to be jaundiced.23 Typical clinical features are nonspecific and include dull right upper quadrant pain, weight loss, and an elevated alkaline phosphatase level.23 Alternatively, asymptomatic patients can present with incidentally detected lesions, when imaging is obtained as part of the workup for other causes or during screening for hepatocellular carcinoma in patients with viral hepatitis or cirrhosis.23,26 Uncommonly, BTC patients present because of signs or symptoms related to metastatic disease or evidence of metastatic disease on imaging.

 

 

Pathology and Grading

The majority of BTCs are adenocarcinomas, corresponding to 90% of cholangiocarcinomas and 99% of gallbladder cancers.27,28 They are graded as well, moderately, or poorly differentiated.2 Adenosquamous and squamous cell carcinoma are responsible for most of the remaining cases.2,29 Cholangiocarcinomas are divided into 3 types, defined by the Liver Cancer Study Group of Japan: (1) mass-forming, (2) periductal-infiltrating, and (3) intraductal-growing.30,31 Mass-forming intrahepatic cholangiocarcinomas are characterized morphologically by a homogeneous gray-yellow mass with frequent satellite nodules and irregular but well-defined margins.17,30 Central necrosis and fibrosis are also common.30 In the periductal-infiltrating type, tumor typically grows along the bile duct wall without mass formation, resulting in concentric mural thickening and proximal biliary dilation.30 Intraductal-growing papillary cholangiocarcinoma is characterized by the presence of intraluminal papillary or tubular polypoid tumors of the intra- or extrahepatic bile ducts, with partial obstruction and proximal biliary dilation.30

Cholangiocarcinoma

Case Presentation

A previously healthy 59-year-old man presents to his primary care physician with a 3-month history of dull right upper quadrant pain associated with weight loss. The patient is markedly cachectic and abdominal examination reveals upper quadrant tenderness. Laboratory exams are significant for elevated alkaline phosphatase (500 U/L; reference range 45–115 U/L), cancer antigen 19-9 (CA 19-9, 73 U/mL; reference range ≤ 37 U/mL), and carcinoembryonic antigen (CEA , 20 ng/mL; reference range for nonsmokers ≤ 3.0 ng/mL). Aspartate aminotransferase, alanine aminotransferase, total bilirubin, and coagulation studies are within normal range. Ultrasound demonstrates a homogeneous mass with irregular borders in the right lobe of the liver. Triphasic contrast-enhanced CT scan demonstrates a tumor with ragged rim enhancement at the periphery, and portal venous phase shows gradual centripetal enhancement of the tumor with capsular retraction. No abdominal lymph nodes or extrahepatic tumors are noted (Figure 1, Image A).

  • What are the next diagnostic steps?

The most critical differential diagnosis of solid liver mass in patients without cirrhosis is cholangiocarcinoma and metastases from another primary site.32 Alternatively, when an intrahepatic lesion is noted on an imaging study in the setting of cirrhosis, the next diagnostic step is differentiation between cholangiocarcinoma and hepatocellular carcinoma (HCC).32 Triphasic contrast-enhanced CT and dynamic magnetic resonance imaging (MRI) are key diagnostic procedures.32,33 In the appropriate setting, classical imaging features in the arterial phase with washout in portal venous or delayed phase can be diagnostic of HCC and may obviate the need for a biopsy (Figure 2).

Typical radiographic features of cholangiocarcinoma include a hypodense hepatic lesion that can be either well-defined or infiltrative and is frequently associated with biliary dilatation (Figure 1, Image A).33 The dense fibrotic nature of the tumor may cause capsular retraction, which is seen in up to 20% of cases.17 This finding is highly suggestive of cholangiocarcinoma and is rarely present in HCC.33 Following contrast administration, there is peripheral (rim) enhancement throughout both arterial and venous phases.32–34 However, these classic features were present in only 70% of cases in one study.35 Although intrahepatic cholangiocarcinomas are most commonly hypovascular, small mass-forming intrahepatic cholangiocarcinomas can often be arterially hyperenhancing and mimic HCC.33 Tumor enhancement on delayed CT imaging has been correlated with survival. Asayama et al demonstrated that tumors that exhibited delayed enhancement on CT in more than two-thirds of their volume were associated with a worse prognosis.36

Patients without cirrhosis who present with a localized lesion of the liver should undergo extensive evaluation for a primary cancer site.37 CT of the chest, abdomen, and pelvis with contrast should be obtained.37 Additionally, mammogram and endoscopic evaluation with esophagogastroduodenoscopy (EGD) and colonoscopy should be included in the work-up.37

Preoperative tumor markers are also included in the work-up. All patients with a solid liver lesion should have serum alpha-fetoprotein (AFP) levels checked. AFP is a serum glycoprotein recognized as a marker for HCC and is reported to detect preclinical HCC.38 However, serum concentrations are normal in up to 40% of small HCCs.38 Although no specific marker for cholangiocarcinoma has yet been identified, the presence of certain tumor markers in the serum of patients may be of diagnostic value, especially in patients with PSC. CA 19-9 and CEA are the best studied. Elevated levels of CA 19-9 prior to treatment are associated with a poorer prognosis, and CA 19-9 concentrations greater than 1000 U/mL are consistent with advanced disease.39,40 One large series evaluated the diagnostic value of serum CEA levels in 333 patients with PSC, 13% of whom were diagnosed with cholangiocarcinoma.34 A serum CEA level greater than 5.2 ng/mL had a sensitivity of 68.0% and specificity of 81.5%.38

If a biopsy is obtained, appropriate immunohistochemistry (IHC) can facilitate the diagnosis. BTC is strongly positive for CK-7 and CK-19.41 CK-7 positivity is not specific and is also common among metastatic cancers of the lung and breast; therefore, in some cases cholangiocarcinoma may be a diagnosis of exclusion. Immunostaining for monoclonal CEA is diffusely positive in up to 75% of cases.41 An IHC panel consisting of Hep Par-1, arginase-1, monoclonal CEA, CK-7, CK-20, TTF-1, MOC-31, and CDX-2 has been proposed to optimize the differential diagnosis of HCC, metastatic adenocarcinoma, and cholangiocarcinoma.41

 

 

Case Continued

CT of the chest, abdomen, and pelvis reveals no concerns for metastasis and no evidence of primary cancer elsewhere. EGD and colonoscopy are clear. AFP levels are within normal limits (2 ng/mL). Biopsy is performed and demonstrates adenocarcinoma. IHC studies demonstrate cells positive for monoclonal CEA, CK-7, CK-19, and MOC-31, and negative for Napsin A, TTF-1, and CK-20.

  • How is cholangiocarcinoma staged and classified?

The purpose of the staging system is to provide information on prognosis and guidance for therapy. Prognostic factors and the therapeutic approaches for BTC differ depending upon their location in the biliary tree. Accordingly, TNM classification systems for intrahepatic, hilar, and distal cholangiocarcinoma and gallbladder cancer have been separated (Table 1 and Table 2).23

For all the subtypes, T stage is mainly dependent upon invasion of adjacent structures rather than size. For perihilar tumors, N category has been reclassified in the newest version of the American Joint Committee on Cancer/Union for International Cancer Control (AJCC/UICC) staging system based upon the number of involved lymph nodes rather than location.

The Bismuth-Corlette classification is used to further classify perihilar cholangiocarcinoma according to patterns of hepatic duct involvement. Type I tumors are located below the confluence of the left and right hepatic ducts.42 Type II reach the confluence of the hepatic ducts.42 Type III occlude the common hepatic duct and either the right or left hepatic duct (IIIa and IIIb, respectively).42 Finally, type IV are multicentric, or involve the confluence and both the right and left hepatic ducts.42 Tumors that involve the common hepatic duct bifurcation are named Klatskin tumors.42

  • What is the first-line treatment for localized cholangiocarcinomas?

Surgical resection is the only potentially curative treatment for localized cholangiocarcinoma, although fewer than 20% of patients are suitable for curative treatment, due to the presence of advanced disease at diagnosis.43,44 Available evidence supports the recommendation that resection with negative margins, regardless of extent, should be the goal of therapy for patients with potentially resectable disease.44 Extensive hepatic resections are often necessary to achieve clear margins since the majority of patients present with large masses. Substantial evidence corroborates that R0 resection is associated with better survival, whereas the benefit of wide compared to narrow (< 5–10 mm) margins is unclear.45 A recent analysis of 96 patients suggests that the proximal resection margin has more prognostic implications than distal margins.45

Surgical options and resectability criteria depend upon tumor location. Extent of tumor in the bile duct is one of the most important factors that determine resectability.17 Although multifocal liver tumors (including satellite lesions), lymph node metastases to the porta hepatis, and distant metastases are considered relative contraindications to surgery, surgical approaches can be considered in selected patients.43 Patient selection for surgery is facilitated by careful preoperative staging, which may include laparoscopy. Laparoscopic staging prior to resection may prevent unnecessary laparotomy in 30% to 45% of patients.42,46

  • Is there a role for adjuvant treatment?

Recurrence following complete resection is a primary limitation for cure in BTC, which provides a rationale for the use of adjuvant therapy.47,48 In a sample of 79 patients with extrahepatic cholangiocarcinoma who underwent curative resection, the cumulative recurrence rate after 4 years was 56%.47 Initial recurrence at a distant site occurs in 40% to 50% of patients.48

Lymphovascular and perineural invasion, lymph node metastasis, and tumor size ≥ 5 cm have been reported as independent predictors of recurrence and mortality following resection.49 A 2017 meta-analysis which included 30 studies involving more than 22,499 patients reported a 41% reduction in the risk of death with adjuvant chemotherapy, which translated to a mean OS benefit of 4 months in an unselected population.49 Moreover, this study revealed inferior OS in patients given adjuvant radiation therapy (RT) in combination with chemotherapy.49 These results are in line with the previous meta-analysis by Horgan et al, which demonstrated that adjuvant RT seems to benefit only patients with R1 resections, with a possible detrimental effect in R0 disease.50 Therefore, adjuvant chemoradiation cannot be viewed as a standard practice following R0 resection, and should be reserved for those patients with positive margins (R1/ 2) to reduce local progression.

In the phase 3 BILCAP trial presented at ASCO 2017, 447 patients with completely resected cholangiocarcinoma or gallbladder cancer with adequate biliary drainage and Eastern Cooperative Oncology Group (ECOG) performance score ≤ 2 were randomly assigned to observation or capecitabine (1250 mg/m2 twice daily for days 1–14 every 21 days for 8 cycles).51 Surgical treatment achieved R0 resection in 62% of patients and 46% were node-negative. Median OS was 51 months for the capecitabine group and 36 months for the control arm (hazard ratio [HR] 0.80, 95% CI 0.63 to 1.04, P = 0.097). Analyses with adjustment for nodal status, grade of disease, and gender indicated a HR of 0.71 (P < 0.01). Median DFS was 25 months versus 18 months favoring the capecitabine group, and rates of grade 3 or 4 toxicity were less than anticipated. Following the results of this trial, adjuvant capecitabine should become the new standard of care.

 

 

  • What is the treatment for locally advanced cholangiocarcinoma?

The optimal approach to patients with locally advanced unresectable cholangiocarcinoma has not been established. The prognosis for patients with either locally unresectable or locally recurrent disease is typically measured in months. Goals of palliative therapy are relief of symptoms and improvement in quality of life, and there is no role for surgical debulking.

Liver transplantation is a potentially curative option for selected patients with hilar or intrahepatic cholangiocarcinoma. Patients with lymph node-negative, non-disseminated, locally advanced hilar cholangiocarcinomas have 5-year survival rates ranging from 25% to 42% following transplantation.52 Retrospective data suggests that neoadjuvant chemoradiation followed by liver transplantation is highly effective for selected patients with hilar cholangiocarcinoma.52 However, these results require confirmation from prospective clinical evidence. It is important to recognize that liver transplantation plays no role in the management of distal cholangiocarcinoma or gallbladder cancer.

Rarely, patients with borderline resectable intrahepatic cholangiocarcinoma will have a sufficient response to chemotherapy to permit later resection, and, in such cases, starting with chemotherapy and then restaging to evaluate resectability is appropriate.54 A single-center, retrospective analysis including 186 patients by Le Roy et al evaluated survival in patients with locally advanced, unresectable intrahepatic cholangiocarcinoma who received primary chemotherapy, followed by surgery in those with secondary resectability.54 After a median of 6 cycles of chemotherapy, 53% of patients achieved resectability and underwent surgery with curative intent. These patients had similar short- and long-term results compared to patients with initially resectable intrahepatic cholangiocarcinoma who had surgery alone, with median OS reaching 24 months.54

Ablative radiotherapy is an additional option for localized inoperable intrahepatic cholangiocarcinoma. Tao and colleagues evaluated 79 consecutive patients with inoperable intrahepatic cholangiocarcinoma treated with definitive RT.55 Median tumor size was 7.9 cm and 89% of patients received chemotherapy before RT. Median OS was 30 months and 3-year OS was 44%. Radiation dose was the single most important prognostic factor, and higher doses correlated with improved local control and OS. A biologic equivalent dose (BED) greater than 80.5 Gy was identified as an ablative dose of RT for large intrahepatic cholangiocarcinomas. The 3-year OS for patients receiving BED greater than 80.5 Gy was 73% versus 38% for those receiving lower doses.

Case Continued

The patient is deemed to have resectable disease and undergoes surgical resection followed by adjuvant capecitabine for 8 cycles. Unfortunately, after 1 year, follow-up imaging identifies bilateral enlarging lung nodules. Biopsy is performed and confirms metastatic cholangiocarcinoma.

  • What is the treatment for metastatic BTC?

The prognosis of patients with advanced BTC is poor and OS for those undergoing supportive care alone is short. A benefit of chemotherapy over best supportive care for cholangiocarcinoma was demonstrated in an early phase 3 trial that randomly assigned 90 patients with advanced pancreatic or biliary cancer (37 with bile duct cancer) to receive either fluorouracil (FU) -based systemic chemotherapy or best supportive care. Results showed that chemotherapy significantly improved OS (6 months versus 2.5 months).56 Chemotherapy is also beneficial for patients with unresectable gallbladder cancer. In a single-center randomized study including 81 patients with unresectable gallbladder cancer, gemcitabine and oxaliplatin (GEMOX) improved progression-free survival (PFS) and OS compared to best supportive care.57 Treatment for metastatic cholangiocarcinoma and gallbladder cancer follows the same algorithm.

In 2010, cisplatin plus gemcitabine was established as a reference regimen for first-line therapy by the ABC-02 study, in which 410 patients with locally advanced or metastatic bile duct, gallbladder, or ampullary cancer were randomly assigned to 6 courses of cisplatin (25 mg/m2) plus gemcitabine (1000 mg/m2 on days 1 and 8, every 21 days) or gemcitabine alone (1000 mg/m2 days 1, 8, 15, every 28 days).58 OS was significantly greater with combination therapy (11.7 versus 8.1 months), and PFS also favored the combination arm (8 versus 5 months). Toxicity was comparable in both groups, with the exception of significantly higher rates of grade 3 or 4 neutropenia with gemcitabine plus cisplatin (25% versus 17%), and higher rates of grade 3 or 4 abnormal liver function with gemcitabine alone (27% versus 17%). Most quality-of-life scales showed a trend favoring combined therapy.58 A smaller, identically designed Japanese phase 3 randomized trial achieved similar results, demonstrating greater OS with cisplatin plus gemcitabine compared to gemcitabine alone (11.2 versus 7.7 months).59

The gemcitabine plus cisplatin combination has not been directly compared with other gemcitabine combinations in phase 3 trials. A pooled analysis of 104 trials of a variety of chemotherapy regimens in advanced biliary cancer concluded that the gemcitabine plus cisplatin regimen offered the highest rates of objective response and tumor control compared with either gemcitabine-free or cisplatin-free regimens.60 However, this did not translate into significant benefit in terms of either time to tumor progression or median OS. It is important to note that this analysis did not include results of the subsequent ABC-02 trial.

There is no standard treatment for patients with cholangiocarcinoma for whom first-line gemcitabine-based therapy fails. There are no completed prospective phase 3 trials supporting the use of second-line chemotherapy after failure of first-line chemotherapy in BTC, and the selection of candidates for second-line therapy as well as the optimal regimen are not established.61 The ongoing phase 2 multicenter ABC-06 trial is evaluating oxaliplatin plus short-term infusional FU and leucovorin (FOLFOX) versus best supportive care for second-line therapy. In a systematic review including 23 studies (14 phase 2 clinical trials and 9 retrospective studies) with 761 patients with BTC, the median OS was 7.2 months.

The optimal selection of candidates for second-line chemotherapy is not established. Two independent studies suggest that patients who have a good performance status (0 or 1), disease control with the first-line chemotherapy, low CA 19-9 level, and possibly previous surgery on their primary tumor, have the longest survival with second-line chemotherapy. However, whether these characteristics predict for chemotherapy responsiveness or more favorable biologic behavior is not clear.62,63 No particular regimen has proved superior to any other, and the choice of second-line regimen remains empiric.

For patients with adequate performance status, examples of other conventional chemotherapy regimens with demonstrated activity that could be considered for second-line therapy include: FOLFOX or capecitabine, gemcitabine plus capecitabine, capecitabine plus cisplatin, or irinotecan plus short-term infusional FU and leucovorin (FOLFIRI) with or without bevacizumab.64 For selected patients, second-line molecularly targeted therapy using erlotinib plus bevacizumab may be considered. However, this regimen is very costly.64 Examples of other regimens with demonstrated activity in phase 2 trials include GEMOX, gemcitabine plus fluoropyrimidine, and fluoropyrimidine plus oxaliplatin or cisplatin.64

There is promising data from studies of targeted therapy for specific molecular subgroups. A recent phase 2 trial evaluated the activity of BGJ398, an orally bioavailable, selective, ATP-competitive pan inhibitor of human fibroblast growth factor receptor (FGFR) kinase, in patients with FGFR-altered advanced cholangiocarcinoma.65 The overall response rate was 14.8% (18.8% FGFR2 fusions only) and disease control rate was 75.4% (83.3% FGFR2 fusions only). All responsive tumors contained FGFR2 fusions. Adverse events were manageable, and grade 3 or 4 treatment-related adverse events occurred in 25 patients (41%). Those included hyperphosphatemia, stomatitis, and palmar-plantar erythrodysesthesia. Javle and colleagues also identified HER2/neu blockade as a promising treatment strategy for gallbladder cancer patients with this gene amplification.66 This retrospective analysis included 9 patients with gallbladder cancer and 5 patients with cholangiocarcinoma who received HER2/neu-directed therapy (trastuzumab, lapatinib, or pertuzumab). In the gallbladder cancer group, HER2/neu gene amplification or overexpression was detected in 8 cases. These patients experienced disease stability (n = 3), partial response (n = 4), or complete response (n = 1) with HER2/neu–directed therapy. Median duration of response was 40 weeks. The cholangiocarcinoma cases treated in this series had no radiological responses despite HER2/neu mutations or amplification.

 

 

Gallbladder Cancer

Case Presentation

A 57-year-old woman from Chile presents with a 3-week history of progressive right upper quadrant abdominal pain. She denies nausea, vomiting, dysphagia, odynophagia, alterations in bowel habits, fever, or jaundice. Her past medical history is significant for obesity and hypertension. She has no history of smoking, alcohol, or illicit drug use. Laboratory studies show marked leukocytosis (23,800/µL) with neutrophilia (91%). Liver function test results are within normal limits. Ultrasound of the abdomen reveals gallbladder wall thickening and cholelithiasis.

The patient undergoes an uneventful laparoscopic cholecystectomy and is discharged from the hospital after 48 hours. Pathology report reveals a moderately differentiated adenocarcinoma of the gallbladder invading the perimuscular connective tissue (T2). No lymph nodes are identified in the specimen.

  • What is the appropriate surgical management of gallbladder cancer?

Gallbladder cancer can be diagnosed preoperatively or can be found incidentally by intraoperative or pathological findings. In one large series, gallbladder cancer was incidentally found during 0.25% of laparoscopic cholecystectomies.67

For patients who are diagnosed with previously unsuspected gallbladder cancer by pathology findings, the extent of tumor invasion (T stage) indicates the need for re-resection (Figure 3).64

Surgical exploration and re-resection are recommended if disease is stage T1b (involving the muscular layer) or higher (Table 2).64,68 In these patients, re-resection is associated with significantly improved OS.68 Patients found to have incidental T1a tumors with negative margins are generally felt to be curable with simple cholecystectomy, and re-resection for T1a tumors does not appear to provide an OS benefit.69,70 The majority of patients diagnosed under these circumstances have T2 or higher disease, and will ultimately require additional surgical exploration.71 A German series that analyzed 439 cases of incidentally diagnosed gallbladder cancer demonstrated that positive lymph nodes were found in 21% and 44% of the re-resected patients with T2 and T3 tumors, respectively.71 There is retrospective data suggesting that the optimal timing of the reoperation is between 4 and 8 weeks following the initial cholecystectomy.72 This interval is believed to be ideal, as it allows for reduced inflammation and does not permit too much time for disease dissemination.72

Alternatively, when gallbladder cancer is documented or suspected preoperatively, adequate imaging is important to identify patients with absolute contraindications to resection. Contraindications to surgery include metastasis, extensive involvement of the hepatoduodenal ligament, encasement of major vessels, and involvement of celiac, peripancreatic, periduodenal, or superior mesenteric nodes.72 Notwithstanding, retrospective series suggest individual patients may benefit, and surgical indications in advanced disease should be determined on an individual basis.73 Staging imaging should be obtained using multiphasic contrast-enhanced CT or MRI of the chest, abdomen, and pelvis. PET-scan can be used in selected cases where metastatic disease is suspected.64 Laparoscopic diagnostic staging should be considered prior to resection.64 This procedure can identify previously unknown contraindications to tumor resection in as much as 23% of patients, and the yield is significantly higher in locally advanced tumors.73

Patients with a diagnosis of potentially resectable, localized gallbladder cancer should be offered definitive surgery. Extended cholecystectomy is recommended for patients stage T2 or above. This procedure involves wedge resection of the gallbladder bed or a segmentectomy IVb/V and lymph node dissection, which should include the cystic duct, common bile duct, posterior superior pancreaticoduodenal lymph nodes, and those around the hepatoduodenal ligament.72 Bile duct excision should be performed if there is malignant involvement.64

Conclusion

BTCs are anatomically and clinically heterogeneous tumors. Prognostic factors and therapeutic approaches for BTCs differ depending upon their location in the biliary tree and, accordingly, TNM classification systems for intrahepatic, hilar, and distal cholangiocarcinoma and gallbladder cancer have been separated. Surgical resection is the only potentially curative treatment for localized BTC. However, recurrence following complete resection is a primary limitation for cure, which provides a rationale for the use of adjuvant therapy. The prognosis of patients with advanced BTC is poor and OS for those undergoing supportive care alone is short. Multiple randomized clinical trials have demonstrated a benefit of chemotherapy for metastatic disease. For patients with adequate performance status, second-line therapy can be considered, and data from studies that evaluated targeted therapy for specific molecular subgroups is promising.

Introduction

Biliary tract carcinoma (BTC) is the term for a heterogeneous group of rare gastrointestinal malignancies1 that includes both carcinoma arising from the gallbladder and cholangiocarcinoma, which refers to diverse aggressive epithelial cancers involving the intrahepatic, perihilar, and distal biliary tree.1–3 In this article, we review the epidemiology, clinical features, and diagnostic approach to BTC, with a focus on current evidence-based treatment strategies for localized, locally advanced, and metastatic BTC.

Epidemiology

In the United States, BTC is rare and accounts for approximately 4% of all gastrointestinal malignancies, with an estimated 6000 to 7000 cases of carcinoma of the gallbladder and 3000 to 4000 cases of carcinoma of the bile duct diagnosed annually.4 Among women, there is a 26-fold variation in BTC mortality worldwide, ranging from 0.8 deaths per 100,000 in South Africa to 21.2 per 100,000 in Chile.1,5 Interestingly, for American Indians in New Mexico, gallbladder cancer mortality rates (8.9 per 100,000) surpass those for breast and pancreatic cancers.6 The incidence of anatomical cholangiocarcinoma subtypes also varies regionally, reflecting disparities in genetic and environmental predisposing factors.2,7 In a large, single-center study in the United States, intrahepatic cholangiocarcinoma accounted for less than 10% of cases, perihilar accounted for 50%, and distal accounted for the remaining 40%.8 Importantly, intrahepatic cholangiocarcinoma is the second most common primary malignancy of the liver, and its incidence seems to be rising in many western countries. In the United States, there has been an estimated 128% rise over the past 40 years.4,9

BTC is associated with high mortality rates.10 Median overall survival (OS) for cholangiocarcinoma is 20 to 28 months and 5-year survival is around 25%.10 Most cholangiocarcinomas are diagnosed at advanced stages with unresectable tumors.10 Furthermore, outcomes following resection with curative intent are poor—median disease-free survival (DFS) of 12 to 36 months has been reported.11,12 Patients with intrahepatic disease have a better prognosis when compared with patients who have extrahepatic tumors.12 Gallbladder cancer, likewise, carries a poor overall prognosis; median OS is 32 months and 5-year survival is as low as 13%.6

Risk factors for BTC include intrinsic and extrinsic elements.6 Incidence of BTC increases with age, and diagnosis typically occurs in the sixth to eighth decade of life.5,6,13 In contrast to gallbladder cancer, the incidence of cholangiocarcinoma is slightly higher in men.9 Obesity, diabetes, and consumption of sweetened drinks also increase the risk for BTC.14–16 Cholelithiasis is the most prevalent risk factor for gallbladder cancer, and the risk is greater for larger stones.5 Around 1 in 5 patients with porcelain gallbladder will develop gallbladder carcinoma.17 Primary sclerosing cholangitis (PSC), chronic calculi of the bile duct, choledochal cysts, cirrhosis, hepatitis C, and liver fluke infections are well established risk factors for cholangiocarcinoma.7,12,18 PSC is one of the best described entities among these predisposing conditions. Lifetime prevalence of cholangiocarcinoma among patients with PSC ranges from 5% to 10%.18,19 These patients also present at a younger age; in one series, the median age at diagnosis for BTC arising from PSC was 39 years.18 It is important to recognize, however, that in most patients diagnosed with cholangiocarcinoma, no predisposing factors are identified.8

Diagnosis

Clinical Presentation

Clinical presentation of BTC depends upon anatomic location.20 Patients with early invasive gallbladder cancer are most often asymptomatic.21 When symptoms occur, they may be nonspecific and mimic cholelithiasis.21 The most common clinical presentations include jaundice, weight loss, and abdominal pain.21 Prior to widespread availability of imaging studies, the preoperative diagnosis rate for gallbladder cancer was as low as 10%.22 However, the accuracy of computed tomography (CT) has changed this scenario, with sensitivity ranging from 73% to 87% and specificity from 88% to 100%.21 As a result of its silent clinical character, cholangiocarcinoma is frequently difficult to diagnose.23 Perihilar and distal cholangiocarcinoma characteristically present with signs of biliary obstruction, and imaging and laboratory data can corroborate the presence of cholestasis.24 On examination, patients with extrahepatic cholangiocarcinoma may present with jaundice, hepatomegaly, and a palpable right upper quadrant mass.25 A palpable gallbladder (Courvoisier sign) can also be present.25 Intrahepatic cholangiocarcinoma presents differently, and patients are less likely to be jaundiced.23 Typical clinical features are nonspecific and include dull right upper quadrant pain, weight loss, and an elevated alkaline phosphatase level.23 Alternatively, asymptomatic patients can present with incidentally detected lesions, when imaging is obtained as part of the workup for other causes or during screening for hepatocellular carcinoma in patients with viral hepatitis or cirrhosis.23,26 Uncommonly, BTC patients present because of signs or symptoms related to metastatic disease or evidence of metastatic disease on imaging.

 

 

Pathology and Grading

The majority of BTCs are adenocarcinomas, corresponding to 90% of cholangiocarcinomas and 99% of gallbladder cancers.27,28 They are graded as well, moderately, or poorly differentiated.2 Adenosquamous and squamous cell carcinoma are responsible for most of the remaining cases.2,29 Cholangiocarcinomas are divided into 3 types, defined by the Liver Cancer Study Group of Japan: (1) mass-forming, (2) periductal-infiltrating, and (3) intraductal-growing.30,31 Mass-forming intrahepatic cholangiocarcinomas are characterized morphologically by a homogeneous gray-yellow mass with frequent satellite nodules and irregular but well-defined margins.17,30 Central necrosis and fibrosis are also common.30 In the periductal-infiltrating type, tumor typically grows along the bile duct wall without mass formation, resulting in concentric mural thickening and proximal biliary dilation.30 Intraductal-growing papillary cholangiocarcinoma is characterized by the presence of intraluminal papillary or tubular polypoid tumors of the intra- or extrahepatic bile ducts, with partial obstruction and proximal biliary dilation.30

Cholangiocarcinoma

Case Presentation

A previously healthy 59-year-old man presents to his primary care physician with a 3-month history of dull right upper quadrant pain associated with weight loss. The patient is markedly cachectic and abdominal examination reveals upper quadrant tenderness. Laboratory exams are significant for elevated alkaline phosphatase (500 U/L; reference range 45–115 U/L), cancer antigen 19-9 (CA 19-9, 73 U/mL; reference range ≤ 37 U/mL), and carcinoembryonic antigen (CEA , 20 ng/mL; reference range for nonsmokers ≤ 3.0 ng/mL). Aspartate aminotransferase, alanine aminotransferase, total bilirubin, and coagulation studies are within normal range. Ultrasound demonstrates a homogeneous mass with irregular borders in the right lobe of the liver. Triphasic contrast-enhanced CT scan demonstrates a tumor with ragged rim enhancement at the periphery, and portal venous phase shows gradual centripetal enhancement of the tumor with capsular retraction. No abdominal lymph nodes or extrahepatic tumors are noted (Figure 1, Image A).

  • What are the next diagnostic steps?

The most critical differential diagnosis of solid liver mass in patients without cirrhosis is cholangiocarcinoma and metastases from another primary site.32 Alternatively, when an intrahepatic lesion is noted on an imaging study in the setting of cirrhosis, the next diagnostic step is differentiation between cholangiocarcinoma and hepatocellular carcinoma (HCC).32 Triphasic contrast-enhanced CT and dynamic magnetic resonance imaging (MRI) are key diagnostic procedures.32,33 In the appropriate setting, classical imaging features in the arterial phase with washout in portal venous or delayed phase can be diagnostic of HCC and may obviate the need for a biopsy (Figure 2).

Typical radiographic features of cholangiocarcinoma include a hypodense hepatic lesion that can be either well-defined or infiltrative and is frequently associated with biliary dilatation (Figure 1, Image A).33 The dense fibrotic nature of the tumor may cause capsular retraction, which is seen in up to 20% of cases.17 This finding is highly suggestive of cholangiocarcinoma and is rarely present in HCC.33 Following contrast administration, there is peripheral (rim) enhancement throughout both arterial and venous phases.32–34 However, these classic features were present in only 70% of cases in one study.35 Although intrahepatic cholangiocarcinomas are most commonly hypovascular, small mass-forming intrahepatic cholangiocarcinomas can often be arterially hyperenhancing and mimic HCC.33 Tumor enhancement on delayed CT imaging has been correlated with survival. Asayama et al demonstrated that tumors that exhibited delayed enhancement on CT in more than two-thirds of their volume were associated with a worse prognosis.36

Patients without cirrhosis who present with a localized lesion of the liver should undergo extensive evaluation for a primary cancer site.37 CT of the chest, abdomen, and pelvis with contrast should be obtained.37 Additionally, mammogram and endoscopic evaluation with esophagogastroduodenoscopy (EGD) and colonoscopy should be included in the work-up.37

Preoperative tumor markers are also included in the work-up. All patients with a solid liver lesion should have serum alpha-fetoprotein (AFP) levels checked. AFP is a serum glycoprotein recognized as a marker for HCC and is reported to detect preclinical HCC.38 However, serum concentrations are normal in up to 40% of small HCCs.38 Although no specific marker for cholangiocarcinoma has yet been identified, the presence of certain tumor markers in the serum of patients may be of diagnostic value, especially in patients with PSC. CA 19-9 and CEA are the best studied. Elevated levels of CA 19-9 prior to treatment are associated with a poorer prognosis, and CA 19-9 concentrations greater than 1000 U/mL are consistent with advanced disease.39,40 One large series evaluated the diagnostic value of serum CEA levels in 333 patients with PSC, 13% of whom were diagnosed with cholangiocarcinoma.34 A serum CEA level greater than 5.2 ng/mL had a sensitivity of 68.0% and specificity of 81.5%.38

If a biopsy is obtained, appropriate immunohistochemistry (IHC) can facilitate the diagnosis. BTC is strongly positive for CK-7 and CK-19.41 CK-7 positivity is not specific and is also common among metastatic cancers of the lung and breast; therefore, in some cases cholangiocarcinoma may be a diagnosis of exclusion. Immunostaining for monoclonal CEA is diffusely positive in up to 75% of cases.41 An IHC panel consisting of Hep Par-1, arginase-1, monoclonal CEA, CK-7, CK-20, TTF-1, MOC-31, and CDX-2 has been proposed to optimize the differential diagnosis of HCC, metastatic adenocarcinoma, and cholangiocarcinoma.41

 

 

Case Continued

CT of the chest, abdomen, and pelvis reveals no concerns for metastasis and no evidence of primary cancer elsewhere. EGD and colonoscopy are clear. AFP levels are within normal limits (2 ng/mL). Biopsy is performed and demonstrates adenocarcinoma. IHC studies demonstrate cells positive for monoclonal CEA, CK-7, CK-19, and MOC-31, and negative for Napsin A, TTF-1, and CK-20.

  • How is cholangiocarcinoma staged and classified?

The purpose of the staging system is to provide information on prognosis and guidance for therapy. Prognostic factors and the therapeutic approaches for BTC differ depending upon their location in the biliary tree. Accordingly, TNM classification systems for intrahepatic, hilar, and distal cholangiocarcinoma and gallbladder cancer have been separated (Table 1 and Table 2).23

For all the subtypes, T stage is mainly dependent upon invasion of adjacent structures rather than size. For perihilar tumors, N category has been reclassified in the newest version of the American Joint Committee on Cancer/Union for International Cancer Control (AJCC/UICC) staging system based upon the number of involved lymph nodes rather than location.

The Bismuth-Corlette classification is used to further classify perihilar cholangiocarcinoma according to patterns of hepatic duct involvement. Type I tumors are located below the confluence of the left and right hepatic ducts.42 Type II reach the confluence of the hepatic ducts.42 Type III occlude the common hepatic duct and either the right or left hepatic duct (IIIa and IIIb, respectively).42 Finally, type IV are multicentric, or involve the confluence and both the right and left hepatic ducts.42 Tumors that involve the common hepatic duct bifurcation are named Klatskin tumors.42

  • What is the first-line treatment for localized cholangiocarcinomas?

Surgical resection is the only potentially curative treatment for localized cholangiocarcinoma, although fewer than 20% of patients are suitable for curative treatment, due to the presence of advanced disease at diagnosis.43,44 Available evidence supports the recommendation that resection with negative margins, regardless of extent, should be the goal of therapy for patients with potentially resectable disease.44 Extensive hepatic resections are often necessary to achieve clear margins since the majority of patients present with large masses. Substantial evidence corroborates that R0 resection is associated with better survival, whereas the benefit of wide compared to narrow (< 5–10 mm) margins is unclear.45 A recent analysis of 96 patients suggests that the proximal resection margin has more prognostic implications than distal margins.45

Surgical options and resectability criteria depend upon tumor location. Extent of tumor in the bile duct is one of the most important factors that determine resectability.17 Although multifocal liver tumors (including satellite lesions), lymph node metastases to the porta hepatis, and distant metastases are considered relative contraindications to surgery, surgical approaches can be considered in selected patients.43 Patient selection for surgery is facilitated by careful preoperative staging, which may include laparoscopy. Laparoscopic staging prior to resection may prevent unnecessary laparotomy in 30% to 45% of patients.42,46

  • Is there a role for adjuvant treatment?

Recurrence following complete resection is a primary limitation for cure in BTC, which provides a rationale for the use of adjuvant therapy.47,48 In a sample of 79 patients with extrahepatic cholangiocarcinoma who underwent curative resection, the cumulative recurrence rate after 4 years was 56%.47 Initial recurrence at a distant site occurs in 40% to 50% of patients.48

Lymphovascular and perineural invasion, lymph node metastasis, and tumor size ≥ 5 cm have been reported as independent predictors of recurrence and mortality following resection.49 A 2017 meta-analysis which included 30 studies involving more than 22,499 patients reported a 41% reduction in the risk of death with adjuvant chemotherapy, which translated to a mean OS benefit of 4 months in an unselected population.49 Moreover, this study revealed inferior OS in patients given adjuvant radiation therapy (RT) in combination with chemotherapy.49 These results are in line with the previous meta-analysis by Horgan et al, which demonstrated that adjuvant RT seems to benefit only patients with R1 resections, with a possible detrimental effect in R0 disease.50 Therefore, adjuvant chemoradiation cannot be viewed as a standard practice following R0 resection, and should be reserved for those patients with positive margins (R1/ 2) to reduce local progression.

In the phase 3 BILCAP trial presented at ASCO 2017, 447 patients with completely resected cholangiocarcinoma or gallbladder cancer with adequate biliary drainage and Eastern Cooperative Oncology Group (ECOG) performance score ≤ 2 were randomly assigned to observation or capecitabine (1250 mg/m2 twice daily for days 1–14 every 21 days for 8 cycles).51 Surgical treatment achieved R0 resection in 62% of patients and 46% were node-negative. Median OS was 51 months for the capecitabine group and 36 months for the control arm (hazard ratio [HR] 0.80, 95% CI 0.63 to 1.04, P = 0.097). Analyses with adjustment for nodal status, grade of disease, and gender indicated a HR of 0.71 (P < 0.01). Median DFS was 25 months versus 18 months favoring the capecitabine group, and rates of grade 3 or 4 toxicity were less than anticipated. Following the results of this trial, adjuvant capecitabine should become the new standard of care.

 

 

  • What is the treatment for locally advanced cholangiocarcinoma?

The optimal approach to patients with locally advanced unresectable cholangiocarcinoma has not been established. The prognosis for patients with either locally unresectable or locally recurrent disease is typically measured in months. Goals of palliative therapy are relief of symptoms and improvement in quality of life, and there is no role for surgical debulking.

Liver transplantation is a potentially curative option for selected patients with hilar or intrahepatic cholangiocarcinoma. Patients with lymph node-negative, non-disseminated, locally advanced hilar cholangiocarcinomas have 5-year survival rates ranging from 25% to 42% following transplantation.52 Retrospective data suggests that neoadjuvant chemoradiation followed by liver transplantation is highly effective for selected patients with hilar cholangiocarcinoma.52 However, these results require confirmation from prospective clinical evidence. It is important to recognize that liver transplantation plays no role in the management of distal cholangiocarcinoma or gallbladder cancer.

Rarely, patients with borderline resectable intrahepatic cholangiocarcinoma will have a sufficient response to chemotherapy to permit later resection, and, in such cases, starting with chemotherapy and then restaging to evaluate resectability is appropriate.54 A single-center, retrospective analysis including 186 patients by Le Roy et al evaluated survival in patients with locally advanced, unresectable intrahepatic cholangiocarcinoma who received primary chemotherapy, followed by surgery in those with secondary resectability.54 After a median of 6 cycles of chemotherapy, 53% of patients achieved resectability and underwent surgery with curative intent. These patients had similar short- and long-term results compared to patients with initially resectable intrahepatic cholangiocarcinoma who had surgery alone, with median OS reaching 24 months.54

Ablative radiotherapy is an additional option for localized inoperable intrahepatic cholangiocarcinoma. Tao and colleagues evaluated 79 consecutive patients with inoperable intrahepatic cholangiocarcinoma treated with definitive RT.55 Median tumor size was 7.9 cm and 89% of patients received chemotherapy before RT. Median OS was 30 months and 3-year OS was 44%. Radiation dose was the single most important prognostic factor, and higher doses correlated with improved local control and OS. A biologic equivalent dose (BED) greater than 80.5 Gy was identified as an ablative dose of RT for large intrahepatic cholangiocarcinomas. The 3-year OS for patients receiving BED greater than 80.5 Gy was 73% versus 38% for those receiving lower doses.

Case Continued

The patient is deemed to have resectable disease and undergoes surgical resection followed by adjuvant capecitabine for 8 cycles. Unfortunately, after 1 year, follow-up imaging identifies bilateral enlarging lung nodules. Biopsy is performed and confirms metastatic cholangiocarcinoma.

  • What is the treatment for metastatic BTC?

The prognosis of patients with advanced BTC is poor and OS for those undergoing supportive care alone is short. A benefit of chemotherapy over best supportive care for cholangiocarcinoma was demonstrated in an early phase 3 trial that randomly assigned 90 patients with advanced pancreatic or biliary cancer (37 with bile duct cancer) to receive either fluorouracil (FU) -based systemic chemotherapy or best supportive care. Results showed that chemotherapy significantly improved OS (6 months versus 2.5 months).56 Chemotherapy is also beneficial for patients with unresectable gallbladder cancer. In a single-center randomized study including 81 patients with unresectable gallbladder cancer, gemcitabine and oxaliplatin (GEMOX) improved progression-free survival (PFS) and OS compared to best supportive care.57 Treatment for metastatic cholangiocarcinoma and gallbladder cancer follows the same algorithm.

In 2010, cisplatin plus gemcitabine was established as a reference regimen for first-line therapy by the ABC-02 study, in which 410 patients with locally advanced or metastatic bile duct, gallbladder, or ampullary cancer were randomly assigned to 6 courses of cisplatin (25 mg/m2) plus gemcitabine (1000 mg/m2 on days 1 and 8, every 21 days) or gemcitabine alone (1000 mg/m2 days 1, 8, 15, every 28 days).58 OS was significantly greater with combination therapy (11.7 versus 8.1 months), and PFS also favored the combination arm (8 versus 5 months). Toxicity was comparable in both groups, with the exception of significantly higher rates of grade 3 or 4 neutropenia with gemcitabine plus cisplatin (25% versus 17%), and higher rates of grade 3 or 4 abnormal liver function with gemcitabine alone (27% versus 17%). Most quality-of-life scales showed a trend favoring combined therapy.58 A smaller, identically designed Japanese phase 3 randomized trial achieved similar results, demonstrating greater OS with cisplatin plus gemcitabine compared to gemcitabine alone (11.2 versus 7.7 months).59

The gemcitabine plus cisplatin combination has not been directly compared with other gemcitabine combinations in phase 3 trials. A pooled analysis of 104 trials of a variety of chemotherapy regimens in advanced biliary cancer concluded that the gemcitabine plus cisplatin regimen offered the highest rates of objective response and tumor control compared with either gemcitabine-free or cisplatin-free regimens.60 However, this did not translate into significant benefit in terms of either time to tumor progression or median OS. It is important to note that this analysis did not include results of the subsequent ABC-02 trial.

There is no standard treatment for patients with cholangiocarcinoma for whom first-line gemcitabine-based therapy fails. There are no completed prospective phase 3 trials supporting the use of second-line chemotherapy after failure of first-line chemotherapy in BTC, and the selection of candidates for second-line therapy as well as the optimal regimen are not established.61 The ongoing phase 2 multicenter ABC-06 trial is evaluating oxaliplatin plus short-term infusional FU and leucovorin (FOLFOX) versus best supportive care for second-line therapy. In a systematic review including 23 studies (14 phase 2 clinical trials and 9 retrospective studies) with 761 patients with BTC, the median OS was 7.2 months.

The optimal selection of candidates for second-line chemotherapy is not established. Two independent studies suggest that patients who have a good performance status (0 or 1), disease control with the first-line chemotherapy, low CA 19-9 level, and possibly previous surgery on their primary tumor, have the longest survival with second-line chemotherapy. However, whether these characteristics predict for chemotherapy responsiveness or more favorable biologic behavior is not clear.62,63 No particular regimen has proved superior to any other, and the choice of second-line regimen remains empiric.

For patients with adequate performance status, examples of other conventional chemotherapy regimens with demonstrated activity that could be considered for second-line therapy include: FOLFOX or capecitabine, gemcitabine plus capecitabine, capecitabine plus cisplatin, or irinotecan plus short-term infusional FU and leucovorin (FOLFIRI) with or without bevacizumab.64 For selected patients, second-line molecularly targeted therapy using erlotinib plus bevacizumab may be considered. However, this regimen is very costly.64 Examples of other regimens with demonstrated activity in phase 2 trials include GEMOX, gemcitabine plus fluoropyrimidine, and fluoropyrimidine plus oxaliplatin or cisplatin.64

There is promising data from studies of targeted therapy for specific molecular subgroups. A recent phase 2 trial evaluated the activity of BGJ398, an orally bioavailable, selective, ATP-competitive pan inhibitor of human fibroblast growth factor receptor (FGFR) kinase, in patients with FGFR-altered advanced cholangiocarcinoma.65 The overall response rate was 14.8% (18.8% FGFR2 fusions only) and disease control rate was 75.4% (83.3% FGFR2 fusions only). All responsive tumors contained FGFR2 fusions. Adverse events were manageable, and grade 3 or 4 treatment-related adverse events occurred in 25 patients (41%). Those included hyperphosphatemia, stomatitis, and palmar-plantar erythrodysesthesia. Javle and colleagues also identified HER2/neu blockade as a promising treatment strategy for gallbladder cancer patients with this gene amplification.66 This retrospective analysis included 9 patients with gallbladder cancer and 5 patients with cholangiocarcinoma who received HER2/neu-directed therapy (trastuzumab, lapatinib, or pertuzumab). In the gallbladder cancer group, HER2/neu gene amplification or overexpression was detected in 8 cases. These patients experienced disease stability (n = 3), partial response (n = 4), or complete response (n = 1) with HER2/neu–directed therapy. Median duration of response was 40 weeks. The cholangiocarcinoma cases treated in this series had no radiological responses despite HER2/neu mutations or amplification.

 

 

Gallbladder Cancer

Case Presentation

A 57-year-old woman from Chile presents with a 3-week history of progressive right upper quadrant abdominal pain. She denies nausea, vomiting, dysphagia, odynophagia, alterations in bowel habits, fever, or jaundice. Her past medical history is significant for obesity and hypertension. She has no history of smoking, alcohol, or illicit drug use. Laboratory studies show marked leukocytosis (23,800/µL) with neutrophilia (91%). Liver function test results are within normal limits. Ultrasound of the abdomen reveals gallbladder wall thickening and cholelithiasis.

The patient undergoes an uneventful laparoscopic cholecystectomy and is discharged from the hospital after 48 hours. Pathology report reveals a moderately differentiated adenocarcinoma of the gallbladder invading the perimuscular connective tissue (T2). No lymph nodes are identified in the specimen.

  • What is the appropriate surgical management of gallbladder cancer?

Gallbladder cancer can be diagnosed preoperatively or can be found incidentally by intraoperative or pathological findings. In one large series, gallbladder cancer was incidentally found during 0.25% of laparoscopic cholecystectomies.67

For patients who are diagnosed with previously unsuspected gallbladder cancer by pathology findings, the extent of tumor invasion (T stage) indicates the need for re-resection (Figure 3).64

Surgical exploration and re-resection are recommended if disease is stage T1b (involving the muscular layer) or higher (Table 2).64,68 In these patients, re-resection is associated with significantly improved OS.68 Patients found to have incidental T1a tumors with negative margins are generally felt to be curable with simple cholecystectomy, and re-resection for T1a tumors does not appear to provide an OS benefit.69,70 The majority of patients diagnosed under these circumstances have T2 or higher disease, and will ultimately require additional surgical exploration.71 A German series that analyzed 439 cases of incidentally diagnosed gallbladder cancer demonstrated that positive lymph nodes were found in 21% and 44% of the re-resected patients with T2 and T3 tumors, respectively.71 There is retrospective data suggesting that the optimal timing of the reoperation is between 4 and 8 weeks following the initial cholecystectomy.72 This interval is believed to be ideal, as it allows for reduced inflammation and does not permit too much time for disease dissemination.72

Alternatively, when gallbladder cancer is documented or suspected preoperatively, adequate imaging is important to identify patients with absolute contraindications to resection. Contraindications to surgery include metastasis, extensive involvement of the hepatoduodenal ligament, encasement of major vessels, and involvement of celiac, peripancreatic, periduodenal, or superior mesenteric nodes.72 Notwithstanding, retrospective series suggest individual patients may benefit, and surgical indications in advanced disease should be determined on an individual basis.73 Staging imaging should be obtained using multiphasic contrast-enhanced CT or MRI of the chest, abdomen, and pelvis. PET-scan can be used in selected cases where metastatic disease is suspected.64 Laparoscopic diagnostic staging should be considered prior to resection.64 This procedure can identify previously unknown contraindications to tumor resection in as much as 23% of patients, and the yield is significantly higher in locally advanced tumors.73

Patients with a diagnosis of potentially resectable, localized gallbladder cancer should be offered definitive surgery. Extended cholecystectomy is recommended for patients stage T2 or above. This procedure involves wedge resection of the gallbladder bed or a segmentectomy IVb/V and lymph node dissection, which should include the cystic duct, common bile duct, posterior superior pancreaticoduodenal lymph nodes, and those around the hepatoduodenal ligament.72 Bile duct excision should be performed if there is malignant involvement.64

Conclusion

BTCs are anatomically and clinically heterogeneous tumors. Prognostic factors and therapeutic approaches for BTCs differ depending upon their location in the biliary tree and, accordingly, TNM classification systems for intrahepatic, hilar, and distal cholangiocarcinoma and gallbladder cancer have been separated. Surgical resection is the only potentially curative treatment for localized BTC. However, recurrence following complete resection is a primary limitation for cure, which provides a rationale for the use of adjuvant therapy. The prognosis of patients with advanced BTC is poor and OS for those undergoing supportive care alone is short. Multiple randomized clinical trials have demonstrated a benefit of chemotherapy for metastatic disease. For patients with adequate performance status, second-line therapy can be considered, and data from studies that evaluated targeted therapy for specific molecular subgroups is promising.

References

1. Goldstein D, Lemech C, Valle J. New molecular and immunotherapeutic approaches in biliary cancer. ESMO Open 2017;2(Suppl 1):e000152.

2. Rizvi S, Khan SA, Hallemeier CL, et al. Cholangiocarcinoma - evolving concepts and therapeutic strategies. Nat Rev Clin Oncol 2017 Oct 10. doi: 10.1038/nrclinonc.2017.157.

3. Hezel AF, Zhu AX. Systemic therapy for biliary tract cancers. Oncologist 2008;13:415–23.

4. U.S. Cancer Statistics Working Group. United States Cancer Statistics: 1999-2014 Incidence and Mortality Web-based Report. Atlanta: U.S. Department of Health and Human Services, Centers for Disease Control and Prevention and National Cancer Institute; 2017.

5. Torre LA, Siegel RL, Islami F, et al. Worldwide burden of and trends in mortality from gallbladder and other biliary tract cancers. Clin Gastroenterol Hepatol 2017 Aug 18. doi: 10.1016/j.cgh.2017.08.017.

6. Lau CSM, Zywot A, Mahendraraj K, Chamberlain CS. Gallbladder carcinoma in the United States: a population based clinical outcomes study involving 22,343 patients from the Surveillance, Epidemiology, and End Result Database (1973–2013). HPB Surg 2017;2017:1532835. doi:10.1155/2017/1532835.

7. Hughes T, O’Connor T, Techasen A, et al. Opisthorchiasis and cholangiocarcinoma in Southeast Asia: an unresolved problem. Int J Gen Med 2017;10:227–37.

8. DeOliveira ML, Cunningham SC, Cameron JL, et al. Cholangiocarcinoma: thirty-one-year experience with 564 patients at a single institution. Ann Surg 2007;245:755–62.

9. Saha SK, Zhu AX, Fuchs CS, Brooks GA. Forty-year trends in cholangiocarcinoma incidence in the U.S.: intrahepatic disease on the rise. Oncologist 2016;21:594–9.

10. Yao KJ, Jabbour S, Parekh N, et al. Increasing mortality in the United States from cholangiocarcinoma: an analysis of the National Center for Health Statistics Database. BMC Gastroenterol 2016;16:117.

11. Choi SB, Kim KS, Choi JY, et al. The prognosis and survival outcome of intrahepatic cholangiocarcinoma following surgical resection: association of lymph node metastasis and lymph node dissection with survival. Ann Surg Oncol 2009;16:3048–56.

12. Endo I, Gonen M, Yopp AC, et al. Intrahepatic cholangiocarcinoma: rising frequency, improved survival, and determinants of outcome after resection. Ann Surg 2008;248:84–96.

13. Duffy A, Capanu M, Abou-Alfa GK, et al. Gallbladder cancer (GBC): 10-year experience at Memorial Sloan-Kettering Cancer Centre (MSKCC). J Surg Oncol 2008;98:485–9.

14. Lauby-Secretan B, Scoccianti C, Loomis D, et al. Body fatness and cancer — viewpoint of the IARC Working Group. N Engl J Med 2016;375:794–8.

15. Chen J, Han Y, Xu C, et al. Effect of type 2 diabetes mellitus on the risk for hepatocellular carcinoma in chronic liver diseases. Eur J Cancer Prev 2015;24:89–99.

16. Larsson SC, Giovannucci EL, Wolk A. Sweetened beverage consumption and risk of biliary tract and gallbladder cancer in a prospective study. J Natl Cancer Inst 2016;108: doi: 10.1093/jnci/djw125.

17. Gore RM. Biliary tract neoplasms: diagnosis and staging. Cancer Imaging 2007;7(Special Issue A):S15–23.

18. Broome U, Olsson R, Lööf L, et al. Natural history and prognostic factors in 305 Swedish patients with primary sclerosing cholangitis. Gut 1996;38:610–5.

19. Burak K, Angulo P, Pasha T, et al. Incidence and risk factors for cholangiocarcinoma in primary sclerosing cholangitis. Am J Gastroenterol 2004;99:523–6.

20. Rodrigues J, Diehl DL. Cholangiocarcinoma: clinical manifestations and diagnosis. Tech Gastrointest Endosc 2016;18:75–82.

21. Mitchell CH, Johnson PT, Fishman EK, et al. Features suggestive of gallbladder malignancy. J Comput Assist Tomogr 2014;38:235–41.

22. Beltz WR, Condon RE. Primary carcinoma of the gallbladder. Ann Surg 1974;180:180–4.

23. Blechacz B, Komuta M, Roskams T, Gores GJ. Clinical diagnosis and staging of cholangiocarcinoma. Nat Rev Gastroenterol Hepatol 2011;8:512–22.

24. Patel T. Cholangiocarcinoma—controversies and challenges. Nat Rev Gastroenterol Hepatol 2011;8:189–200.

25. Nakeeb A, Pitt HA, Sohn TA, et al. Cholangiocarcinoma. A spectrum of intrahepatic, perihilar, and distal tumors. Ann Surg 1996;224:463–73.

26. Bartella I, Dufour JF. Clinical diagnosis and staging of intrahepatic cholangiocarcinoma. J Gastrointestin Liver Dis 2015;24:481-9.

27. Yamaguchi K, Enjoji M. Carcinoma of the gallbladder: a clinicopathology of 103 patients and a newly proposed staging. Cancer 1988;62:1425–32.

28. Esposito I, Schirmacher P. Pathological aspects of cholangiocarcinoma. HPB. 2008;10:83–6.

29. Silva VWK, Askan G, Daniel TD, et al. Biliary carcinomas: pathology and the role of DNA mismatch repair deficiency. Chin Clin Oncol 2016;5:62.

30. Chung YE, Kim MJ, Park YN, et al. Varying appearances of cholangiocarcinoma: radiologic-pathologic correlation. Radiographics 2009;29:683–700.

31. Yamasaki S. Intrahepatic cholangiocarcinoma: macroscopic type and stage classification. J Hepatobiliary Pancreat Surg 2003;10:288–91.

32. Rao PN. Nodule in liver: investigations, differential diagnosis and follow-up. J Clin Exp Hepatol 2014;4(Suppl 3):S57–62.

33. Kim TK, Lee E, Jang HJ. Imaging findings of mimickers of hepatocellular carcinoma. Clin Mol Hepatol 2015;21:326–43.

34. Hennedige TP, Neo WT, Venkatesh SK. Imaging of malignancies of the biliary tract- an update. Cancer Imaging 2014;14:14.

35. Kim SH, Lee CH, Kim BH, et al. Typical and atypical imaging findings of intrahepatic cholangiocarcinoma using gadolinium ethoxybenzyl diethylenetriamine pentaacetic acid-enhanced magnetic resonance imaging. J Comput Assist Tomogr 2012;36:704–9.

36. Asayama Y, Yoshimitsu K, Irie H, et al. Delayed-phase dynamic CT enhancement as a prognostic factor for mass-forming intrahepatic cholangiocarcinoma. Radiology 2006;238:150–5.

37. National Comprehensive Cancer Network. Cancer of unknown primary. www.nccn.org/professionals/physician_gls/pdf/bone.pdf. Accessed 1 Dec 2017.

38. Kefeli A, Basyigit S, Yeniova AO. Diagnosis of hepatocellular carcinoma. In: Abdeldayem HM, ed. Updates in liver cancer. London: InTech; 2017.

39. Bergquist JR, Ivanics T, Storlie CB, et al. Implications of CA19-9 elevation for survival, staging, and treatment sequencing in intrahepatic cholangiocarcinoma: A national cohort analysis. J Surg Oncol 2016;114:475–82.

40. Chung YJ, Choi DW, Choi SH, et al. Prognostic factors following surgical resection of distal bile duct cancer. J Korean Surg Soc 2013;85:212–8.

41. Lau SK, Prakash S, Geller SA, Alsabeh R. Comparative immunohistochemical profile of hepatocellular carcinoma, cholangiocarcinoma, and metastatic adenocarcinoma. Hum Pathol 2002;33:1175–81.

42. Paul A, Kaiser GM, Molmenti EP, et al. Klatskin tumors and the accuracy of the Bismuth-Corlette classification. Am Surg 2011;77:1695–9.

43. Cannavale A, Santoni M, Gazzetti M, et al. Updated management of malignant biliary tract tumors: an illustrative review. J Vasc Interv Radiol 2016;27:1056–69.

44. Matsuo K, Rocha FG, Ito K, et al. The Blumgart preoperative staging system for hilar cholangiocarcinoma: analysis of resectability and outcomes in 380 patients. J Am Coll Surg 2012;215:343–55.

45. Yoo T, Park SJ, Han SS, et al. Proximal resection margins: more prognostic than distal resection margins in patients undergoing hilar cholangiocarcinoma resection. Cancer Res Treat 2017 Nov 16; doi.org/10.4143/crt.2017.320.

46. Joseph S, Connor S, Garden OJ. Staging laparoscopy for cholangiocarcinoma. HPB 2008;10:116–9.

47. Jarnagin WR, Ruo L, Little SA, et al. Patterns of initial disease recurrence after resection of gallbladder carcinoma and hilar cholangiocarcinoma: implications for adjuvant therapeutic strategies. Cancer 2003;98:1689–700.

48. Kobayashi A, Miwa S, Nakata T, Miyagawa S. Disease recurrence patterns after R0 resection of hilar cholangiocarcinoma. Br J Surg 2010;97:56–64.

49. Ghidini M, Tomasello G, Botticelli A, et al. Adjuvant chemotherapy for resected biliary tract cancers: a systematic review and meta-analysis. HPB 2017;19:741–8.

50. Horgan AM, Amir E, Walter T, Knox JJ. Adjuvant therapy in the treatment of biliary tract cancer: a systematic review and meta-analysis. J Clin Oncol 2012;30:1934–40.

51. Primrose JN, Fox R, Palmer DH, et al. Adjuvant capecitabine for biliary tract cancer: the BILCAP randomized study [abstract]. J Clin Oncol 2017 35:15_suppl:4006-4006. 

52. Darwish Murad S, Kim WR, Darnois DM, et al. Efficacy of neoadjuvant chemoradiation followed by liver transplantation for perihilar cholangiocarcinoma at 12 US centers. Gastroenterology 2012;143:88–98.

53. Sapisochin G, Facciuto M, Rubbia-Brandt L, et al. Liver transplantation for “very early” intrahepatic cholangiocarcinoma: International retrospective study supporting a prospective assessment. Hepatology 2016;64:1178–88.

54. Le Roy B, Gelli M, Pittau G, et al. Neoadjuvant chemotherapy for initially unresectable intrahepatic cholangiocarcinoma. Br J Surg 2017 Aug 31. doi: 10.1002/bjs.10641.

55. Tao R, Krishnan S, Bhosale PR, et al. Ablative radiotherapy doses lead to a substantial prolongation of survival in patients with inoperable intrahepatic cholangiocarcinoma: a retrospective dose response analysis. J Clin Oncol 2016;34:219–26.

56. Glimelius B, Hoffman K, SjÓdén PO, et al. 555 Palliative chemotherapy improves survival and quality of life in advanced pancreatic and biliary cancer. Eur J Cancer 1995;31:S118.

57. Sharma A, Dwary AD, Mohanti BK, et al. Best supportive care compared with chemotherapy for unresectable gall bladder cancer: a randomized controlled study. J Clin Oncol 2010;28:4581–6.

58. Valle J, Wasan H, Palmer DH, et al. Cisplatin plus gemcitabine versus gemcitabine for biliary tract cancer. N Engl J Med 2010;362:1273–81.

59. Okusaka T, Nakachi K, Fukutomi A, et al. Gemcitabine alone or in combination with cisplatin in patients with biliary tract cancer: a comparative multicentre study in Japan. Br J Cancer 2010;103:469–74.

60. Eckel F, Schmid RM. Chemotherapy in advanced biliary tract carcinoma: a pooled analysis of clinical trials. Br J Cancer 2007;96:896–902.

61. Lamarca A, Hubner RA, David Ryder W, Valle JW. Second-line chemotherapy in advanced biliary cancer: a systematic review. Ann Oncol 2014;25:2328–38.

62. Brieau B, Dahan L, De Rycke Y, et al. Second-line chemotherapy for advanced biliary tract cancer after failure of the gemcitabine-platinum combination: A large multicenter study by the Association des Gastro-Entérologues Oncologues. Cancer 2015;121:3290–7.

63. Fornaro L, Cereda S, Aprile G, et al. Multivariate prognostic factors analysis for second-line chemotherapy in advanced biliary tract cancer. Br J Cancer 2014;110:2165–9.

64. National Comprehensive Cancer Network. Hepatobiliary cancer. www.nccn.org/professionals/physician_gls/pdf/hepatobiliary.pdf. Accessed 12 Nov 2017.

65. Javle M, Lowery M, Shroff RT, et al. Phase II study of BGJ398 in patients with FGFR-altered advanced cholangiocarcinoma. J Clin Oncol 2017 Nov 28;JCO2017755009.

66. Javle M, Churi C, Kang HC, et al. HER2/neu-directed therapy for biliary tract cancer. J Hematol Oncol 2015;8:58.

67. Konstantinidis IT, Deshpande V, Genevay M, et al. Trends in presentation and survival for gallbladder cancer during a period of more than 4 decades: a single-institution experience. Arch Surg 2009;144:441–47.

68. Singh S, Agarwal AK. Gallbladder cancer: the role of laparoscopy and radical resection. Ann Surg 2009;250:494–5.

69. Kapoor VK, Haribhakti SP. Extended cholecystectomy for carcinoma of the gall bladder. Trop Gastroenterol 1995;16:74–5.

70. Ethun CG, Postlewait LM, Le N, et al. Association of optimal time Interval to re-resection for incidental gallbladder cancer with overall survival: a multi-Institution analysis from the US extrahepatic biliary malignancy consortium. JAMA Surg 2017;152:143–9.

71. Goetze TO, Paolucci V. Benefits of reoperation of T2 and more advanced incidental gallbladder carcinoma: analysis of the German registry. Ann Surg 2008;247:104–8.

72. Nishio H, Nagino M, Ebata T, et al. Aggressive surgery for stage IV gallbladder carcinoma; what are the contraindications? J Hepatobiliary Pancreat Surg 2007;14:351–7.

73. Agarwal AK, Kalayarasan R, Javed A, et al. The role of staging laparoscopy in primary gallbladder cancer--an analysis of 409 patients: a prospective study to evaluate the role of staging laparoscopy in the management of gallbladder cancer. Ann Surg 2013;258:318–23.

References

1. Goldstein D, Lemech C, Valle J. New molecular and immunotherapeutic approaches in biliary cancer. ESMO Open 2017;2(Suppl 1):e000152.

2. Rizvi S, Khan SA, Hallemeier CL, et al. Cholangiocarcinoma - evolving concepts and therapeutic strategies. Nat Rev Clin Oncol 2017 Oct 10. doi: 10.1038/nrclinonc.2017.157.

3. Hezel AF, Zhu AX. Systemic therapy for biliary tract cancers. Oncologist 2008;13:415–23.

4. U.S. Cancer Statistics Working Group. United States Cancer Statistics: 1999-2014 Incidence and Mortality Web-based Report. Atlanta: U.S. Department of Health and Human Services, Centers for Disease Control and Prevention and National Cancer Institute; 2017.

5. Torre LA, Siegel RL, Islami F, et al. Worldwide burden of and trends in mortality from gallbladder and other biliary tract cancers. Clin Gastroenterol Hepatol 2017 Aug 18. doi: 10.1016/j.cgh.2017.08.017.

6. Lau CSM, Zywot A, Mahendraraj K, Chamberlain CS. Gallbladder carcinoma in the United States: a population based clinical outcomes study involving 22,343 patients from the Surveillance, Epidemiology, and End Result Database (1973–2013). HPB Surg 2017;2017:1532835. doi:10.1155/2017/1532835.

7. Hughes T, O’Connor T, Techasen A, et al. Opisthorchiasis and cholangiocarcinoma in Southeast Asia: an unresolved problem. Int J Gen Med 2017;10:227–37.

8. DeOliveira ML, Cunningham SC, Cameron JL, et al. Cholangiocarcinoma: thirty-one-year experience with 564 patients at a single institution. Ann Surg 2007;245:755–62.

9. Saha SK, Zhu AX, Fuchs CS, Brooks GA. Forty-year trends in cholangiocarcinoma incidence in the U.S.: intrahepatic disease on the rise. Oncologist 2016;21:594–9.

10. Yao KJ, Jabbour S, Parekh N, et al. Increasing mortality in the United States from cholangiocarcinoma: an analysis of the National Center for Health Statistics Database. BMC Gastroenterol 2016;16:117.

11. Choi SB, Kim KS, Choi JY, et al. The prognosis and survival outcome of intrahepatic cholangiocarcinoma following surgical resection: association of lymph node metastasis and lymph node dissection with survival. Ann Surg Oncol 2009;16:3048–56.

12. Endo I, Gonen M, Yopp AC, et al. Intrahepatic cholangiocarcinoma: rising frequency, improved survival, and determinants of outcome after resection. Ann Surg 2008;248:84–96.

13. Duffy A, Capanu M, Abou-Alfa GK, et al. Gallbladder cancer (GBC): 10-year experience at Memorial Sloan-Kettering Cancer Centre (MSKCC). J Surg Oncol 2008;98:485–9.

14. Lauby-Secretan B, Scoccianti C, Loomis D, et al. Body fatness and cancer — viewpoint of the IARC Working Group. N Engl J Med 2016;375:794–8.

15. Chen J, Han Y, Xu C, et al. Effect of type 2 diabetes mellitus on the risk for hepatocellular carcinoma in chronic liver diseases. Eur J Cancer Prev 2015;24:89–99.

16. Larsson SC, Giovannucci EL, Wolk A. Sweetened beverage consumption and risk of biliary tract and gallbladder cancer in a prospective study. J Natl Cancer Inst 2016;108: doi: 10.1093/jnci/djw125.

17. Gore RM. Biliary tract neoplasms: diagnosis and staging. Cancer Imaging 2007;7(Special Issue A):S15–23.

18. Broome U, Olsson R, Lööf L, et al. Natural history and prognostic factors in 305 Swedish patients with primary sclerosing cholangitis. Gut 1996;38:610–5.

19. Burak K, Angulo P, Pasha T, et al. Incidence and risk factors for cholangiocarcinoma in primary sclerosing cholangitis. Am J Gastroenterol 2004;99:523–6.

20. Rodrigues J, Diehl DL. Cholangiocarcinoma: clinical manifestations and diagnosis. Tech Gastrointest Endosc 2016;18:75–82.

21. Mitchell CH, Johnson PT, Fishman EK, et al. Features suggestive of gallbladder malignancy. J Comput Assist Tomogr 2014;38:235–41.

22. Beltz WR, Condon RE. Primary carcinoma of the gallbladder. Ann Surg 1974;180:180–4.

23. Blechacz B, Komuta M, Roskams T, Gores GJ. Clinical diagnosis and staging of cholangiocarcinoma. Nat Rev Gastroenterol Hepatol 2011;8:512–22.

24. Patel T. Cholangiocarcinoma—controversies and challenges. Nat Rev Gastroenterol Hepatol 2011;8:189–200.

25. Nakeeb A, Pitt HA, Sohn TA, et al. Cholangiocarcinoma. A spectrum of intrahepatic, perihilar, and distal tumors. Ann Surg 1996;224:463–73.

26. Bartella I, Dufour JF. Clinical diagnosis and staging of intrahepatic cholangiocarcinoma. J Gastrointestin Liver Dis 2015;24:481-9.

27. Yamaguchi K, Enjoji M. Carcinoma of the gallbladder: a clinicopathology of 103 patients and a newly proposed staging. Cancer 1988;62:1425–32.

28. Esposito I, Schirmacher P. Pathological aspects of cholangiocarcinoma. HPB. 2008;10:83–6.

29. Silva VWK, Askan G, Daniel TD, et al. Biliary carcinomas: pathology and the role of DNA mismatch repair deficiency. Chin Clin Oncol 2016;5:62.

30. Chung YE, Kim MJ, Park YN, et al. Varying appearances of cholangiocarcinoma: radiologic-pathologic correlation. Radiographics 2009;29:683–700.

31. Yamasaki S. Intrahepatic cholangiocarcinoma: macroscopic type and stage classification. J Hepatobiliary Pancreat Surg 2003;10:288–91.

32. Rao PN. Nodule in liver: investigations, differential diagnosis and follow-up. J Clin Exp Hepatol 2014;4(Suppl 3):S57–62.

33. Kim TK, Lee E, Jang HJ. Imaging findings of mimickers of hepatocellular carcinoma. Clin Mol Hepatol 2015;21:326–43.

34. Hennedige TP, Neo WT, Venkatesh SK. Imaging of malignancies of the biliary tract- an update. Cancer Imaging 2014;14:14.

35. Kim SH, Lee CH, Kim BH, et al. Typical and atypical imaging findings of intrahepatic cholangiocarcinoma using gadolinium ethoxybenzyl diethylenetriamine pentaacetic acid-enhanced magnetic resonance imaging. J Comput Assist Tomogr 2012;36:704–9.

36. Asayama Y, Yoshimitsu K, Irie H, et al. Delayed-phase dynamic CT enhancement as a prognostic factor for mass-forming intrahepatic cholangiocarcinoma. Radiology 2006;238:150–5.

37. National Comprehensive Cancer Network. Cancer of unknown primary. www.nccn.org/professionals/physician_gls/pdf/bone.pdf. Accessed 1 Dec 2017.

38. Kefeli A, Basyigit S, Yeniova AO. Diagnosis of hepatocellular carcinoma. In: Abdeldayem HM, ed. Updates in liver cancer. London: InTech; 2017.

39. Bergquist JR, Ivanics T, Storlie CB, et al. Implications of CA19-9 elevation for survival, staging, and treatment sequencing in intrahepatic cholangiocarcinoma: A national cohort analysis. J Surg Oncol 2016;114:475–82.

40. Chung YJ, Choi DW, Choi SH, et al. Prognostic factors following surgical resection of distal bile duct cancer. J Korean Surg Soc 2013;85:212–8.

41. Lau SK, Prakash S, Geller SA, Alsabeh R. Comparative immunohistochemical profile of hepatocellular carcinoma, cholangiocarcinoma, and metastatic adenocarcinoma. Hum Pathol 2002;33:1175–81.

42. Paul A, Kaiser GM, Molmenti EP, et al. Klatskin tumors and the accuracy of the Bismuth-Corlette classification. Am Surg 2011;77:1695–9.

43. Cannavale A, Santoni M, Gazzetti M, et al. Updated management of malignant biliary tract tumors: an illustrative review. J Vasc Interv Radiol 2016;27:1056–69.

44. Matsuo K, Rocha FG, Ito K, et al. The Blumgart preoperative staging system for hilar cholangiocarcinoma: analysis of resectability and outcomes in 380 patients. J Am Coll Surg 2012;215:343–55.

45. Yoo T, Park SJ, Han SS, et al. Proximal resection margins: more prognostic than distal resection margins in patients undergoing hilar cholangiocarcinoma resection. Cancer Res Treat 2017 Nov 16; doi.org/10.4143/crt.2017.320.

46. Joseph S, Connor S, Garden OJ. Staging laparoscopy for cholangiocarcinoma. HPB 2008;10:116–9.

47. Jarnagin WR, Ruo L, Little SA, et al. Patterns of initial disease recurrence after resection of gallbladder carcinoma and hilar cholangiocarcinoma: implications for adjuvant therapeutic strategies. Cancer 2003;98:1689–700.

48. Kobayashi A, Miwa S, Nakata T, Miyagawa S. Disease recurrence patterns after R0 resection of hilar cholangiocarcinoma. Br J Surg 2010;97:56–64.

49. Ghidini M, Tomasello G, Botticelli A, et al. Adjuvant chemotherapy for resected biliary tract cancers: a systematic review and meta-analysis. HPB 2017;19:741–8.

50. Horgan AM, Amir E, Walter T, Knox JJ. Adjuvant therapy in the treatment of biliary tract cancer: a systematic review and meta-analysis. J Clin Oncol 2012;30:1934–40.

51. Primrose JN, Fox R, Palmer DH, et al. Adjuvant capecitabine for biliary tract cancer: the BILCAP randomized study [abstract]. J Clin Oncol 2017 35:15_suppl:4006-4006. 

52. Darwish Murad S, Kim WR, Darnois DM, et al. Efficacy of neoadjuvant chemoradiation followed by liver transplantation for perihilar cholangiocarcinoma at 12 US centers. Gastroenterology 2012;143:88–98.

53. Sapisochin G, Facciuto M, Rubbia-Brandt L, et al. Liver transplantation for “very early” intrahepatic cholangiocarcinoma: International retrospective study supporting a prospective assessment. Hepatology 2016;64:1178–88.

54. Le Roy B, Gelli M, Pittau G, et al. Neoadjuvant chemotherapy for initially unresectable intrahepatic cholangiocarcinoma. Br J Surg 2017 Aug 31. doi: 10.1002/bjs.10641.

55. Tao R, Krishnan S, Bhosale PR, et al. Ablative radiotherapy doses lead to a substantial prolongation of survival in patients with inoperable intrahepatic cholangiocarcinoma: a retrospective dose response analysis. J Clin Oncol 2016;34:219–26.

56. Glimelius B, Hoffman K, SjÓdén PO, et al. 555 Palliative chemotherapy improves survival and quality of life in advanced pancreatic and biliary cancer. Eur J Cancer 1995;31:S118.

57. Sharma A, Dwary AD, Mohanti BK, et al. Best supportive care compared with chemotherapy for unresectable gall bladder cancer: a randomized controlled study. J Clin Oncol 2010;28:4581–6.

58. Valle J, Wasan H, Palmer DH, et al. Cisplatin plus gemcitabine versus gemcitabine for biliary tract cancer. N Engl J Med 2010;362:1273–81.

59. Okusaka T, Nakachi K, Fukutomi A, et al. Gemcitabine alone or in combination with cisplatin in patients with biliary tract cancer: a comparative multicentre study in Japan. Br J Cancer 2010;103:469–74.

60. Eckel F, Schmid RM. Chemotherapy in advanced biliary tract carcinoma: a pooled analysis of clinical trials. Br J Cancer 2007;96:896–902.

61. Lamarca A, Hubner RA, David Ryder W, Valle JW. Second-line chemotherapy in advanced biliary cancer: a systematic review. Ann Oncol 2014;25:2328–38.

62. Brieau B, Dahan L, De Rycke Y, et al. Second-line chemotherapy for advanced biliary tract cancer after failure of the gemcitabine-platinum combination: A large multicenter study by the Association des Gastro-Entérologues Oncologues. Cancer 2015;121:3290–7.

63. Fornaro L, Cereda S, Aprile G, et al. Multivariate prognostic factors analysis for second-line chemotherapy in advanced biliary tract cancer. Br J Cancer 2014;110:2165–9.

64. National Comprehensive Cancer Network. Hepatobiliary cancer. www.nccn.org/professionals/physician_gls/pdf/hepatobiliary.pdf. Accessed 12 Nov 2017.

65. Javle M, Lowery M, Shroff RT, et al. Phase II study of BGJ398 in patients with FGFR-altered advanced cholangiocarcinoma. J Clin Oncol 2017 Nov 28;JCO2017755009.

66. Javle M, Churi C, Kang HC, et al. HER2/neu-directed therapy for biliary tract cancer. J Hematol Oncol 2015;8:58.

67. Konstantinidis IT, Deshpande V, Genevay M, et al. Trends in presentation and survival for gallbladder cancer during a period of more than 4 decades: a single-institution experience. Arch Surg 2009;144:441–47.

68. Singh S, Agarwal AK. Gallbladder cancer: the role of laparoscopy and radical resection. Ann Surg 2009;250:494–5.

69. Kapoor VK, Haribhakti SP. Extended cholecystectomy for carcinoma of the gall bladder. Trop Gastroenterol 1995;16:74–5.

70. Ethun CG, Postlewait LM, Le N, et al. Association of optimal time Interval to re-resection for incidental gallbladder cancer with overall survival: a multi-Institution analysis from the US extrahepatic biliary malignancy consortium. JAMA Surg 2017;152:143–9.

71. Goetze TO, Paolucci V. Benefits of reoperation of T2 and more advanced incidental gallbladder carcinoma: analysis of the German registry. Ann Surg 2008;247:104–8.

72. Nishio H, Nagino M, Ebata T, et al. Aggressive surgery for stage IV gallbladder carcinoma; what are the contraindications? J Hepatobiliary Pancreat Surg 2007;14:351–7.

73. Agarwal AK, Kalayarasan R, Javed A, et al. The role of staging laparoscopy in primary gallbladder cancer--an analysis of 409 patients: a prospective study to evaluate the role of staging laparoscopy in the management of gallbladder cancer. Ann Surg 2013;258:318–23.

Issue
Hospital Physician: Hematology/Oncology - 13(1)
Issue
Hospital Physician: Hematology/Oncology - 13(1)
Publications
Publications
Topics
Article Type
Sections
Citation Override
2018 January/February;13(1):23-35
Disallow All Ads
Content Gating
No Gating (article Unlocked/Free)
Alternative CME
Disqus Comments
Default
Use ProPublica